You are on page 1of 58

Torts Outline Prof. Heymann Fall 2009 Table of Contents Short Outline ............................................................................................................................................ 3 Intentional Torts .......................................................................................................................................

. 10 Prima Facie Case .......................................................................................................................... 10 Battery .............................................................................................................................. 10 Trespass ............................................................................................................................ 12 Conversion ........................................................................................................................ 13 False Imprisonment ........................................................................................................... 15 Assault .............................................................................................................................. 16 Outrage ............................................................................................................................. 17 Privileges ...................................................................................................................................... 19 Defense of Person and Property ......................................................................................... 19 Private Necessity ............................................................................................................... 21 Public Necessity ................................................................................................................ 22 The Negligence Standard .......................................................................................................................... 22 Duty.............................................................................................................................................. 22 The Reasonable Person...................................................................................................... 22 Mental Ability and Mental States ........................................................................... 22 Physical Infirmities ................................................................................................ 23 Age ........................................................................................................................ 24 Duties Arising from Affirmative Acts................................................................................ 25 Duties Arising Undertakings ............................................................................................. 25 Special Relationships ........................................................................................................ 27 Duties Arising from the Occupation of Land ..................................................................... 29 Negligent Infliction of Emotional Distress ......................................................................... 31 Breach .......................................................................................................................................... 34 Hand Formula ................................................................................................................... 34 Custom and Medical Malpractice ...................................................................................... 35 Negligence Per Se ............................................................................................................. 37 Res Ipsa Loquitor .............................................................................................................. 38 Causation ...................................................................................................................................... 41 Page 1 of 58

Torts Outline Prof. Heymann Fall 2009 Cause in Fact ..................................................................................................................... 41 But For Causation .................................................................................................. 41 Alternative Liability............................................................................................... 42 Market Share Liability ........................................................................................... 42 Proximate Cause................................................................................................................ 43 Direct..................................................................................................................... 43 Foreseeability ........................................................................................................ 44 Intervening Causes................................................................................................. 44 Limitations of Liability .......................................................................................... 45 Damages ....................................................................................................................................... 46 Compensatory Damages .................................................................................................... 46 Damages to Property.............................................................................................. 47 Personal Injury/Death ............................................................................................ 47 Pain and Suffering ................................................................................................. 48 Punitive Damages .............................................................................................................. 48 Strict Liability ......................................................................................................................................... 50 Liability for Animals .................................................................................................................... 51 Abnormally Dangerous Activities ................................................................................................ 51 Respondeat Superior .................................................................................................................... 52 Defenses ................................................................................................................................................... 54 Contributory and Comparative Negligence.................................................................................... 55 Express Assumption of Risk ......................................................................................................... 55 Primary and Secondary Assumption of Risk ................................................................................. 57 Primary Assumption of Risk.............................................................................................. 57 Secondary Assumption of the Risk .................................................................................... 58

Page 2 of 58

Torts Outline Prof. Heymann Fall 2009 Short Outline I. Intentional Torts a. Prima Facie Case i. Battery Intent to cause a harmful or offensive contact or apprehension of imminent contact 1. Defenses to Battery a. No specific intent b. Consent if you consent to violation of bodily integrity then its not battery ii. Trespass Intent to Enter anothers land 1. An unintentional and non-negligent entry on land in the possession of another, or causing a thing or third person to enter the land, does not subject the actor to liability to the possessor iii. Trespass to chattels Intent to intermeddle iv. Conversion Intent to exercise domain 1. P does not have to exhaust all means possible to gain possession of the items that were taken - the simple fact that it is withheld is conversion v. False Imprisonment Intent to Confine vi. Assault Intent to cause a harmful or offensive contact or apprehension of imminent contact vii. Outrage Intent to cause severe emotional distress b. Privileges i. Defense of Person and Property 1. Self Defense the use of reasonable non-deadly force on another who is about to inflict intentional harm 2. Defense of a Third Person the use of reasonable non-deadly force to defend a third person from a harmful or offensive contact 3. Defense of Property the use of reasonable force to prevent the commission of a tort against their property ii. Private Necessity a person can commit trespass to a chattel or conversion if the act is or is reasonably believed to be necessary to prevent injury to himself or his property iii. Public Necessity - one can commit trespass to a chattel or a conversion if the act is or is reasonably believed to be necessary for the purpose of avoiding a public disaster Negligence a. Duty i. Generally people have a duty of reasonable care and must act as a reasonable man under like circumstances would 1. Mental Ability and Mental States - D must act as would a person of average mental ability; individual mental handicaps (eg. Low IQ) are not considered except in regards to the P 2. Physical Infirmities - a person is expected to know their physical handicaps and must act with reasonable care under the circumstances 3. Age a. Minors a child is generally held to that measure of care that other minors of like age, experience, capacity and development would ordinarily exercise under similar circumstances; but a minor can be held to an adult Page 3 of 58

II.

Torts Outline Prof. Heymann Fall 2009 standard of care if he is engaging in an activity that is dangerous to others and is normally engaged in by adults b. The elderly - an elderly individual is held to the reasonable person standard; they are not excused from liability because of their age ii. Affirmative Acts 1. A D has no duty to an individual unless he created the risk through his affirmative acts 2. Encouraging or enticing a mentally competent adult into a risky situation is not an affirmative act and thus does not create a duty of care 3. There is no duty to assist in an emergency (when asked) unless there is an imminent threat of bodily injury and the burden is low (remember you have no duty to help others at the risk of your own safety) iii. Undertakings 1. A D may acquire a duty by undertaking to provide assistance or otherwise voluntarily assuming responsibilities 2. Once an individual undertakes to render services they must do so with reasonable care; failure to use reasonable care will result in liability 3. When you begin to render assistance to an individual, you cant discontinue the aid and leave the individual in a position worse than they were before you assumed control iv. Special Relationships 1. Duty to assist others - Common Carriers have a duty of care to assist their passengers and prevent foreseeable injuries that occur while transporting the passenger to a reasonably safe place 2. Duties to third parties a. There is no duty of care to warn an individual of danger/threat except in cases where there is a specific and precise threat from the third party and where a relationship exists between the actor and the third party b. There is a duty to protect an individual from harm of a third party when a D has exclusive control over the premises where an injury occurs c. The police do not have a duty to protect a specific individual unless they can prove a special relationship (eg. Police beginning to render aid through witness protection); the duty extends to public at large 3. Duties Arising from the Occupation of Land a. Trespassers one who comes onto the land without permission or privilege; i. Generally no duty, except duty of reasonable care in carrying out activities risking serious bodily harm for known trespassers (w/ duty to warn of highly dangerous conditions in some jurisdictions) b. Licensee one who enters on the land with the landowners permission, express or implied, for her own purpose or business rather than the landowners benefit; social guests are considered licensees i. Duty to warn of dangerous hidden conditions of which owner is aware (and licensee is unaware) ii. Duty of reasonable care in undertaking activities iii. No duty to inspect Page 4 of 58

Torts Outline Prof. Heymann Fall 2009 c. Invitees one who enters onto the premises in response to an express or implied invitation of the landowner and includes those who enter onto premises opened to the public (eg. Museums) and those connected with the business (eg. Store customers); business visitors are considered invitees i. Duty of reasonable care both as to condition of property and as to activities ii. Duty to warn and duty to inspect 4. Negligent Infliction of Emotional Distress (NIED) - This is not a separate tort; it is a way of talking about a type of cases in which the question is whether the D had a duty to refrain from carelessly causing the emotional distress; the harm is the emotional distress in the negligent case; it is at root a negligence case; there must be a physical manifestation of the mental suffering/injury (eg. Depression or anxiety) a. Zone of physical danger cases are those in which the P is the direct victim of the negligence (eg. near miss cases) b. By-stander cases are those in which someone else is the victim of physical injury and the P witnessed the injury b. Breach failure of the D to comply with their duty i. Hand Formula B = the burden/cost of taking adequate precautions P = probability of the harm occurring as a result of untaken precaution L = the gravity of the resulting injury B>PL = not negligent B<PL = negligent ii. Custom and Malpractice 1. Custom a. Customs help to determine what is considered reasonable care b. Even though a D may take the precautions that are standard or customary in their field, a ct. can rule that the precaution breaches a duty, if the standard is not an example of reasonable care c. Evidence of compliance with custom or non-compliance with an industry custom is relevant and admissible, but not determinative 2. Medical Malpractice a. Medical customs are dispositive and the standard of care a doctor was bound by either falls under the locality or national rule b. Locality rule i. Old Point of View - doctors were bound by the standards that were prevalent in the community in which they practiced ii. New Point of View- doctors should be bound by the standards that are prevalent in communities that have similar geography and medical facilities as the community in which they practice c. National rule Doctors are bound by the standard of care of the average practitioner Page 5 of 58

Torts Outline Prof. Heymann Fall 2009 3. Negligence Per Se when a D violates a statute that creates a standard of care, the violation establishes the D negligence as a matter of law; the statute has to be related to the claim a. Exception to the negligence per se rule - If complying with a statute poses a greater danger to an individual than non-compliance, the individual will not be found liable for violating the statute 4. Res Ipsa Loquitor used to show breach when a P cant point to a particular action a. The thing that happened to the P must be the type of event that ordinarily does not occur except through the negligence of someone b. The instrumentality of the harm suffered by the P must have been w/in the D's exclusive control c. The harm to the P must not have resulted from any conduct on the part of the P c. Causation Ds breach of duty was the cause of the Ps injury i. Cause in fact the Ds conduct was the actual cause of the injury 1. But for test But for the Ds negligent act, the injury would not have occurred; P has the burden to show that the untaken precaution would more likely than not have prevented the injures (51% threshold) 2. Loss Chance of Survival - when a P has difficulty proving that the Ds conduct was the actual cause of the injury, they can use loss chance of survival to argue that Ds conduct decreased their chance to survive and resulted in harm 3. Alternative Liability - if its clear that 2 Ds brought about the negligent situation, both parties will be held liable; D has the burden of showing that the other defendant caused the injury 4. Market Share Liability prevalent in defective product cases where there are three or more Ds; assign liability based on percentage of market share 5. Joint and Several Liability - when it is impossible to tell which D caused part of the injury, the P can collect the all of the damages from either D; has been replaced with apportionment of damages ii. Proximate Causation 1. Proximate cause isnt a separate cause; it limits the chain of liability 2. Direct - D is liable for all consequences that flowed directly from their negligent acts regardless of whether the consequences were foreseeable 3. Foreseeability - D is liable for foreseeable consequences of their negligent acts ; it does not matter if the consequence is a rare occurrence or if the D did not foresee the extent of the damage d. Damages i. Compensatory Damages - P recovers damages to compensate her for the losses proximately resulting from the Ds tortious act or omission. 1. Damage to Property - the damages are measured by the loss in value of the property or the cost to repair (replace), whichever is less. 2. Personal Injury/Death - P is entitled to recover to repair Ps injury or make him as whole as may be done by an award of money can factor in the following: a. Medical expenses Page 6 of 58

Torts Outline Prof. Heymann Fall 2009 b. Loss of consortium/society loss of companionship as a result of a decedents death c. Lost earnings and earning capacity (subtract taxes but add benefits) i. Factors to be considered: 1. Occupation/Education 2. Health/Age 3. Dependents 3. Pain and Suffering Awards - P may be able to use an award of general damages to provide activities or enjoyments that substitute for those lost as a result of the injury ii. Punitive Damages - serve as a punishment to deter a D and other parties from committing a similar tort in the future 1. Punitive damages can be awarded against an employer for the conduct of an employee if but only if: a. The employer authorized the conduct and the manger of the conduct or b. The employee was unfit and the employer was reckless in employing him or c. The employee was employed in a managerial capacity and was acting in the scope of his employment or d. The employer ratified or approved the act Strict Liability - focuses on activities where we say the nature of the activity is such that we want the D to pay whether they're at fault at not. a. Liability for Animals i. A person who keeps an animal with knowledge (scienter retinuit) of its tendency to do harm is strictly liable for the damage that occurs if it escapes; person has an absolute duty to confine or control the animal to prevent injury and liability will result for failure to control ii. Animals that are ferae naturae are animals that are by nature harmful or have not been tamed and domesticated; they are presumed to have a tendency to do harm and scienter doesn't have to be proved iii. Animals that are mansuetae naturae are presumed to be harmless until they manifest a savage or vicious propensity; proof of this manifestation is necessary to determine liability (usually based on animals prior bad acts) b. Abnormally Dangerous Activities - In determining whether an activity is abnormally dangerous, the following factors are to be considered: 1. Existence of a high degree of risk of some harm to the person, land or chattels of others; 2. Likelihood that the harm that results from it will be great; 3. Inability to eliminate the risk by the exercise of reasonable care; 4. Extent to which the activity is not a matter of common usage 5. Inappropriateness of the activity to the place where it is carried on; and 6. Extent to which its value to the community is outweighed by its dangerous attributes c. Respondeat Superior - holds an employer strictly liable for the torts committed by their employees in the course of their work (aka scope of employment) Page 7 of 58

III.

Torts Outline Prof. Heymann Fall 2009 i. Who is an employee? - If the one securing a service (ie the company) controls the means by which the task is accomplished, then the one performing the service is an employee; if not, then they are an independent contractor ii. Frolic and Detour 1. Frolic the employee pursues personal business and this qualifies as a substantial deviation from or abandonment of the employment 2. Detour employees deviation is still sufficiently related to the employment to fall within the scope of employment iii. Commuting to Work - in general, employers arent responsible for torts committed while an employee is commuting to work 1. Exception when the employers conduct creates an extra risk to the public which will manifest itself during the commute (eg. Drinking) Defenses a. Contributory and Comparative Negligence i. Contributory Negligence - D argues that the P failed to take reasonable care in their own safety; this defense if proven, is a complete bar to recovery regardless if D is shown to have some liability 1. Last Clear Chance doctrine - a P could recover despite committing contributory negligence if the D had a sufficiently good opportunity to avoid the accident at a point when the P did not ii. Comparative Negligence 1. Pure Comparative Negligence - reduces the P's damages by the percentage of negligence; permits recovery even if Ps fault is greater than D 2. Modified Comparative Negligence - P's recover based on the percentage if: a. Ps negligence does not exceed the Ds (50% or more) or b. Ps negligence is less than the D's (49% or less) 3. If a P has engaged in unlawful conduct, courts will not entertain suit if Ps conduct constitutes a serious violation of the law and the injuries for which P seeks recovery are the direct result of that violation b. Express Assumption of Risk i. Express Assumption of Risk P explicitly agrees to assume the risk of injury and frees D of liability ii. Questions to ask: 1. Did the P understand what they were agreeing to? 2. Even if they did, should the contract not be enforced because of policy reasons? iii. To enforce the contract there must be: 1. Exculpatory Clauses 2. Equal bargaining power iv. Waivers of liability are unenforceable if they involve public interests c. Primary and Secondary Assumption of Risk i. Primary Assumption of Risk - D had no duty to protect the P from the harm suffered because the risk of it was inherent in an activity that the P chose to undertake; the inherent activity entails certain risks that P is aware of and decides to ignore 1. Determination of liability depends on the following: a. The openness of the risk is it hidden or misrepresented b. the voluntariness of the Ps participation in the activity in question Page 8 of 58

IV.

Torts Outline Prof. Heymann Fall 2009 c. the reasonableness of the risks posed by the activity in light of its benefits ii. Secondary Assumption of Risk - D had a duty to P because the risk was not inherent in the activity but P knowingly and voluntarily chose to encounter the risk (eg. Need to get to the hospital during an emergency and volunteer to get in a car with a drunk driver)

Page 9 of 58

Torts Outline Prof. Heymann Fall 2009 I. Intentional Torts A. The Prima Facie Case i. Battery 1. Elements of Battery a. An actor is subject to liability to another for battery if: i. He acts intending to cause a harmful or offensive contact with the person of the other or a third,and ii. A harmful or offensive contact with the person of the other directly or indirectly results. 2. Intent a. Concerned about the intent to cause contact, not necessarily the intent to cause harm/leaving a mark i. Egg-shell doctrine: the idea that you are liable for damages that directly result from your actions, even if the extent of the damages is unanticipated i. White v. University of Idaho (piano teacher student touched like keyboard)(L): D intended contact but did not intend the extent of such harm

b. Transferred Intent: When A intends to commit a battery against B but mistakenly hits C instead, A can be liable to C i. Manning v. Grimsley (pitcher threw ball at hecklers and hit a third party)(L): in this case, the tort was assault, but the pitchers intent to hit the hecklers and instead hitting a third party is a prime example ii. Keel v. Hamline 3. Harmful or Offensive Contact a. Concerned with whether there has been a violation of the Ps bodily integrity; a mark doesnt have to be left behind for a contact be harmful or offensive i. Leichtman v.WLW Jacor Communications (DJ blows smoke into someones face)(L): Smoke contains particulate matter (some part of it is physical) so it is battery; also if someone tries to argue that it is not physical you could respond saying harm is about violation of bodily integrity and blowing smokes violates b. Contact extends not only to the individuals body but also includes anything in contact with the individual such as clothing, packages held by individual that are snatched in a rude or insolent manner i. Morgan v. Loyacomo - (manager chases suspected shoplifter and snatches bag)(L) Page 10 of 58

Torts Outline Prof. Heymann Fall 2009 c. Contact is offensive if it would offend a reasonable persons sense of dignity but some contact, although unwanted by the individual is reasonable under certain circumstances (eg. Walking down a crowded street and being tapped on the shoulder) i. Wallace v. Rosen (mother pushed down stairs during fire drill by teacher)(NL) 4. Defenses to Battery a. No specific intent i. Madden v. D.C. Transit System, Inc. (buses spewing emissions)(NL): D cannot be held liable since the contact is not directed at anyone in particular b. Consent if you consent to violation of bodily integrity then its not battery i. Implied Consent Considering the surroundings and circumstances, an individuals participation in certain conduct signifies consent i. Knight v. Jewett (boy steps on girls hand during touch football)(NL): D did not intend the contact but injury during the game can be reasonably foreseen thus a persons willingness to participate in such an activity implies that they are willing to take the risk of injury

ii. Vosburg v. Putney (kid kicks classmate)(L): a classroom is a location where one would not expect to be kicked in the shin, thus there is not an implied consent to the kick iii. Keel v. Hainline (kids throwing eraser back and forth in classroom mistakenly hits third party)(L): D did not intend contact but the conduct was unlawful and it occurred in a classroom where there is no implied license ii. Express Consent An individual communicates their consent for contact to another i. If you make your preference known, then doctor should abide by preference a. Cohen v. Smith (woman asks for no man to see her naked but a man sees her)(L): D violated her consent ii. Consent for contact granted to a particular individual cannot be transferred to another w/o the patients approval a. Grabowski v. Quigley (gave consent to one doctor but another did surgery)(L): Ps right to choose physician was taken away while he was unconscious iii. A D cannot be liable for violating consent if they act within the scope of consent given Page 11 of 58

Torts Outline Prof. Heymann Fall 2009 a. Brzoska v. Olson (dentist w/ AIDS)(NL) iv. Informed Consent a. In an emergency situation, a patients prior consent for a type of contact can be dismissed if it was not an informed consent i. Werth v. Taylor (Jehovahs witness refused blood transfusion before her life became endangered)(NL): patients prior refusal to blood transfusion was based on the idea that this was routine surgery and that her life would not be in danger; the situation changed and her prior refusals were not decided based on this new information - it was not an informed consent ii. Trespass 1. Elements of Trespass (Rest. 158, pg. 32) a. One is subject to liability to another for trespass, irrespective of whether he thereby causes harm to any legally protected interest of the other, if he intentionally i. Enters land in the possession of the other, or causes a thing or third person to do so, ii. Remains on the land, or iii. Fails to remove from the land a thing which he is under a duty to remove. 2. Consent to entry is given legal effect even though the entrant is gaining it under misrepresentation or omission and if the real intent is known would provide reasons for owner to revoke his consent i. Desnick v. American Broadcasting Companies, Inc. - (reporters go undercover in eye care center)(NL): Policy arguments for rule i. It is helpful to society (ie food critics and anti-discrimination research)

ii. It is not so invasive that it infringes on the kind of interest the law of trespass protects (no public disturbance, videotaped communications were professional - not personal, the reporters did not invade private space iii. The trespass does not interfere in the ownership or possession of land 3. Non-liability for Accidental intrusions (Rest. 166, pg. 34)

Page 12 of 58

Torts Outline Prof. Heymann Fall 2009 a. An unintentional and non-negligent entry on land in the possession of another, or causing a thing or third person to enter the land, does not subject the actor to liability to the possessor b. Malouf v. Dallas Athletic Country Club (golfers hit balls that enter Ps property and damage cars)(NL): the golfers intended to hit the ball into the hole on the golf course; they did not intend for the ball to stray from the golf course property; w/o the proper intent, they are not liable i. Hypo 1: Let's assume that the property owner has a laundry line that is obscured by trees but you can see it from the golf course. It is a piece of laundry (red T-shirt) flapping away in the breeze. Thinking it's the flag for the hole you hit the ball which hits the car. Is this trespass? golfer still intends to hit the ball and enter in the land so it's trespass; even if they are mistaken as to who owns the land it doesn't matter because they intended to enter the land ii. Hypo 2: same facts as above but the ball smacks into a gas tank on P's property, the tank explodes and the entire house burns down. What damages is the golfer liable in trespass? the gravity of the results doesn't change the basic facts; he is still liable (similar to Vosburg and egg-shell house) c. Pegg v. Gray - (man permit dogs to follow foxes onto neighbors property)(L): Normally a dog owner is not responsible for damages for its entry upon someone else's property since the dog does so of its own volition. In the case of fox hunting, if a dog is released and the owner has knowledge that the dog is likely to go onto someone else's land in pursuit of the fox and the dog owner does not get permission to onto the land beforehand, the dog owner is liable for trespass; he intentionally let the dog roam onto the property iii. Trespass to Chattels and Conversion 1. Trespass to Chattels (D pay for damages; mild version) a. Ways of Committing Trespass to Chattel (Rest. 217, pg. 42) i. A trespass to a chattel may be committed by intentionally i. Dispossessing another of the chattel, or ii. Using or intermeddling with a chattel in the possession of another b. Elements to trespass of chattels (Rest. 218, pg. 42) i. One who commits a trespass to a chattel is subject to liability to the possessor of the chattel if, but only if, i. He dispossesses the other of the chattel, or ii. The chattel is impaired as to its condition, quality, or value, or iii. The possessor is deprived of the use of the chattel for a substantial time, or Page 13 of 58

Torts Outline Prof. Heymann Fall 2009 iv. Bodily harm is caused to the possessor, or harm is caused to some person or thing in which the possessor has a legally protected interest 2. Conversion (D pay for full value; strong version) a. Elements of Conversion (Rest. 222A, pg. 42) i. Conversion is an intentional exercise of dominion or control over a chattel which so SERIOUSLY interferes with the right of another to control it that the actor may justly be required to pay the other the full value of the chattel. ii. In determining the seriousness of the interference and the justice of requiring the actor to pay the full value, the following factors are important: i. The extent and duration of the actors exercise of dominion or control;

ii. The actors intent to assert a right in fact inconsistent with the others right of control; iii. The actors good faith; iv. The extent and duration of the resulting interference with the others right of control; v. The harm done to the chattel; vi. The inconvenience and expense caused to the other b. P does not have to exhaust all means possible to gain possession of the items that were taken - the simple fact that it is withheld is conversion i. Russell-Vaughn Ford, Inc. v. Rouse (car salesman intentionally took keys and refused to return them)(L): Yes, Rouse could have called his wife to get the spare key but he shouldnt have to c. If a person acts outside of the scope of the contract (permitting the use of the item) then they will be held liable regardless of whether the damage was the result of an accident or negligence i. Palmer v. Mayo (Mayo rented horse and carriage from Palmer; Mayo lent carriage to Scott and Cook; Scott and Cook get into accident and wreck carriage and kill horse; Palmer sues Cook)(L): It doesnt matter if the injury was an accident or not, Cook did not receive permission from Palmer to use the horse and carriage and the injury interfered w/ the owners right; Note: conversion happened when Cook took the carriage ii. Spooner v. Manchester (man borrows horse and gets lost on the way home; tries to find right path but horse injured)(NL): D was intending to return the horse as outlined in his contract; the steps he took was with the intent to return the horse as promised Page 14 of 58

Torts Outline Prof. Heymann Fall 2009 i. Hypo: A guy leaves his pickup truck in a parking lot and an imposter calls towing business saying he's owner and asked for towing place to tow it to mechanic. Driver takes the truck and tows it to shop. It's later stolen from the shop by a third party. Owner of truck sues towing company. a. P argument - tow company intentionally took the truck and exerted control over it; the tow interfered w/ owners use b. D argument - I was acting in good faith that you asked me to move the truck c. Despite D's lack of knowledge of ownership, he still intentionally moved something that did not belong to him d. The fact that someone stole the truck from the lot does not have relevance to the conversion and the driver's actions e. The results of this case can be a deterrent because the driver, seeing they can be blamed, could later enact some type of verification procedure iv. False Imprisonment 1. Elements of False Imprisonment (Rest. 35, pg. 56) a. An actor is subject to liability to another for false imprisonment if i. He acts intending to confine the other or a third person within boundaries fixed by the actor, and ii. His act directly or indirectly results in such a confinement of the other, and iii. The other is conscious of the confinement or is harmed by it b. An act which is not done with the intention stated in Subsection (a)(i) does not make the actor liable to the other for a merely transitory or otherwise harmless confinement, although the act involves an unreasonable risk of imposing it and therefore would be negligent or reckless if the risk threatened bodily harm. 2. What constitutes confinement a. To make the actor liable for false imprisonment, the others confinement within the boundaries fixed by the actor must be complete b. The confinement is complete although there is reasonable means of escape, unless the other knows of it

Page 15 of 58

Torts Outline Prof. Heymann Fall 2009 c. The actor does not become liable for false imprisonment by intentionally preventing another from going in a particular direction in which he has a right or privilege to go 3. Intent to Confine a. Hypo: Runaway Bride A woman gets cold feet and places a call that she's been abducted and provides a description of the abductor. The cops pick up a man based on the description. Is she guilty of false imprisonment? although she is not doing the actual confinement, it was not directed at a particular individual so this may lack intent b. Hypo: Bride decides to file a report saying her fianc assaulted her and the police arrest. Is she guilty of false imprisonment? since her actions were directed at her fianc, she is providing information w/ the intent to confine 4. Legal authority to imprison a. A D is liable for false imprisonment even if D believed they had the legal authority to imprison the P i. Bright v. Ailshie (bounty hunter arrested the wrong brother)(L): this is false imprisonment because the bounty hunter intentionally arrested the wrong brother and brother was aware of the confinement; mistaken identity is not a defense to false imprisonment b. A third party can be held liable for false imprisonment if they persuade/encourage the police or some other authority to arrest the individual i. Baggett v. National Bank & Trust Co. (bank customer used deposit slip that claimed this is a stek up and was arrested)(NL): The decision to arrest was made solely by the police w/o a request from the bank that he be detained or held ii. Melton v. LaCalamito (U-haul manager demanded the return of moving pads that did not belong to U-haul)(L): the manager continued to insist over a long out period of time that the pads were U-hauls thus leaving the police w/ no choice but to arrest v. Assault 1. Elements of Assault (Rest. 21, pg. 65) a. An actor is subject to liability to another for assault if i. He acts intending to cause a harmful or offensive contact with the person of the other or a third person, or an imminent apprehension of such a contact, and ii. The other is thereby put in such imminent apprehension. b. An action which is not done with the intention stated in Subsection (a)(i) does not make the actor liable to the other for an apprehension caused thereby although the act involves an unreasonable risk of causing it and, therefore, would be negligent or reckless if the risk threatened bodily harm. Page 16 of 58

Torts Outline Prof. Heymann Fall 2009 2. Similarities and Differences b/t Assault and Battery a. With assault we're focusing on inviability of mental state (when a ball is thrown at you, it is assault until it makes contact; upon contact it is battery) b. Both have question of whether there was an intentional/volitional act c. The result in battery was violation of bodily integrity; in assault it is violation of mental integrity 3. Remember apprehension is not the equivalent of fear, its anticipation 4. Intent Requirement a. Act can prove intent i. Bennight v. Western Auto Supply Co. (employee is attacked by bats in a warehouse, the next day her manger forces her to go into warehouse again and she is bitten)(L): Act provides the intent Manager required her to work in there knowing that she was already attacked once and knowing that the possibility of being bitten existed; apprehension is bolstered by the fact that she was already attacked once ii. Langford v. Shu (mother aided and abetted her children in playing a prank on a woman who was injured in flight)(L): Boys are going to be boys and try to pull pranks but as an adult you should have known better; this applies to the general rule of assault, she knew the woman would be afraid and she still did it anyway 5. Imminent Threat of Harm a. Verbalized threat alone is not enough to be considered an imminent threat of harm i. Brower v. Ackerley (brothers threaten politician over billboard dispute)(NL): As a policy matter you wouldnt want to punish the brothers because people say things in the heat of the moment and it would be a waste of resources to arrest people who just trash talk ii. Tuberville v. Savage (swordsman declares that he wont hurt man because ct. is in session)(NL): swordsmans statement shows that there was no imminent threat since he would restrain himself CANT SUE FOR ASSAULT WHEN THE ACT IS DISCLAIMED vi. Outrage/Intentional Infliction of Emotional Distress (IIED) 1. Elements of Outrage (Rest. 46, pg. 71) a. One who by extreme and outrageous conduct intentionally or recklessly causes severe emotional distress to another is subject to liability for such emotional distress, and if bodily harm to the other results from it, for such bodily harm

Page 17 of 58

Torts Outline Prof. Heymann Fall 2009 b. Where such conduct is directed at a third person, the actor is subject to liability if he intentionally or recklessly causes severe emotional distress i. To a member of such person's immediate family who is present at the time, whether or not such distress results in bodily harm, or ii. To any other person who is present at the time, if such distress results in bodily harm 2. For a while people were afraid of giving validity to the tort for the following reasons (possible policy implications): a. It was difficult to prove b. A lot of people would claim IIED which would bog down the cts. And result in inefficiency c. There are some duties that result in IIED that are legit (ex. Bill collecting) 3. Outrage is a community based standard which must take into account the statement and the attendant circumstances a. Pros provides flexibility when needed b. Cons wishy-washy; could be influenced by prejudice or bias c. Roberts v. Saylor (doctor tells woman he doesnt like her before she goes into surgery; woman tried to get him to testify against other doctors)(NL): Ct. held that given their negative prior history, the doctor was just letting off some steam; civilized society would not deem the doctors behavior has so outrageous that it is intolerable d. Greer v. Medders (doctor screams at his patient (who is lying in the hospital bed) and patients wife after they called his office looking for him)(L): the patient was lying in the hospital bed, recovering from surgery so the doctor should have taken better care; this conduct would be in intolerable to a civilized society 4. If a D knows that P is sensitive and thus more susceptible to emotional distress then they can be held liable a. When a D is sensitive, the conduct, which may not be extreme and outrageous in one situation, would be extreme and outrageous in another situation; eggshell theory b. Muratore v. M/S Scotia Prince (photographers in gorilla suits try take pictures of woman on cruise and when she refuses, they harass her for the remainder of the trip)(L): D knew she had a sensitivity to pictures and continued to harass her and make lewd comments; the degree of their conduct was outrageous 5. The ct. can consider the personality of the individual to whom the conduct is directed a. Pemberton v. Bethlehem Steel Corp. (PI hired by D sent photos from an affair to Ps wife and circulated mug shots to the employees in his Page 18 of 58

Torts Outline Prof. Heymann Fall 2009 union)(NL): the sending of truthful information is not considered outrageous or extreme B. Privileges i. Defense of Person and Property 1. Comes up when P has made the prima facie case and the D is claiming a privilege that excuses the tort 2. Defense of Person a. Self-Defense i. An actor is privileged to use reasonable force, not intended or likely to cause death or serious bodily harm, to defend himself against unprivileged harmful or offensive contact or other bodily harm which he reasonably believes that another is about to inflict intentionally upon him. ii. There is a subjective component (D must believe they are under threat) and objective (D must have reasonable response) iii. Qualifications to self-defense (Rest. 63, pg. 95) i. Actor's duty to avoid force - The actor must cannot reasonably believe that the use of force is necessary until he has exhausted all other reasonably safe means of preventing the other from inflicting bodily harm

ii. Actor's duty to retreat - The actor, if he reasonably believes that he is threatened with the intentional imposition of bodily harm, or even of an offensive contact, may stand his ground and repel the attack by the use of reasonable force, which does not threaten serious harm or death, even though he might with absolute certainty of safety avoid the threatened bodily harm or offensive contact by retreating iii. Standing one's ground in his dwelling place - One attacked in his dwelling place may await his assailant and use deadly force to repel him though he could prevent the assailant from the attacking him by closing the door and so excluding the assailant from the premises. But the mere fact that a man is threatened with an attack while he is within his own dwelling place does not justify him in using deadly weapons if he can avoid the necessity of so doing by an alternative other than flight or standing a siege. A man can no more justify using deadly weapons when he is in his own home than he can when he is upon a public highway, if he can avoid the necessity of doing so by complying with a demand, other than a demand that he shall retreat, give up the possession of his dwelling or Page 19 of 58

Torts Outline Prof. Heymann Fall 2009 permit an intrusion into, it, or abandon an attempt to make a lawful arrest. b. If, in the course of defending himself, an individual accidentally injures a third party then the individual can be by protected self-defense in an intentional tort (mind you they can be liable for negligence) i. Crabtree v. Dawson - (bouncer at club, mistakenly hits a bystander who he believed was going to attack him)(NL): D used reasonable care to repel the threat and honestly believed P was an attacker c. Defense of a Third Person (Rest. 76, pg. 96) i. The actor is privileged to defend a third person from a harmful or offensive contact or other invasion of his interests of personality under the same conditions and by the same means as those under and by which he is privileged to defend himself if the actor correctly or reasonably believes that i. The circumstances are such as to give the third person a privilege of self-defense, and

ii. His intervention is necessary for the protection of the third person 3. Defense of Property a. Generally, one may use reasonable force to prevent the commission of a tort against their property i. Katko v. Briney - (D used a spring gun to protect property after repeated break-ins)(L): the use of a spring gun which cant be controlled was unreasonable and would likely cause death and harm; an individual cannot use such deadly force to protect property ii. Woodbridge v. Marks (watchdogs attack a man who strayed from path)(NL): watchdogs are a means of reasonable force to repel the threat; owner took the proper steps by keeping the dogs away from pathways and the residence; their confinement would only affect trespassers because trespassers (in an attempt to stay hidden) would not use the paths b. Hypo: i. Man constantly yells at kids to get off of his lawn; he gets a gun and shoots in the air; the bullet comes down and hits a kid there is battery but can't claim self-defense because it was deadly force, also how much of a threat can a kid be ii. Same guy is defending himself against an armed robber and he fires but the bullet goes out the window and hits someone on the street privileged to defend himself against robber; absent privilege there would be transferred intent to the unintended victim; robber could be Page 20 of 58

Torts Outline Prof. Heymann Fall 2009 charged w/ assault towards the homeowner and then can transfer intent for the injury to the bystander 4. If D is committing a felony against the P and D injures P, D can argue self-defense a. Wright v. Haffke (grocery store owner shoots thief in the back, as thief tries to escape after robbing store)(NL): the use of a firearm is not justified in minor thefts and can only be used in more serious felonies such as robberies ii. Private Necessity 1. Any person is privileged to commit an act which would otherwise be a trespass to a chattel or conversion if the act is or is reasonably believed to be necessary to prevent injury to himself or his property a. Ploof v. Putnam (family tries to moor ship to escape storm; servant unmoors the vessel and the family is injured/vessel destroyed)(L): The doctrine of necessity applies especially in circumstances where there is a danger to human life so the servant cant argue that mooring the ship is a trespass b. Rossi v. DelDuca - (girl runs onto mans property to escape dog; ends up being attacked by the dogs on the property)(NL): P's pursuit by the first dog granted her the privilege of entering the land (relieving her from liability of trespass) since P was attempting to escape injury and destroys the D's immunity from liability in resisting the intrusion 2. Private necessity can also apply in situations that are not drastically dangerous situations a. Texas Midland Ry. Co. v. Geraldon (RR agent puts family out in the rain even though they said the rain would make the wife sick)(L): Agents knowledge of wifes susceptibility to illness and the fact that the rain would aggravate it, was a threat to her health; family had a privilege to stay on the premises to ensure that she would not suffer an illness/injury 3. Even though D has the right to trespass or convert, they must compensate the P for the actual damage to the Ps property a. Vincent v. Lake Erie Transportation Co. (steamship damages dock when moored in a storm)(L): The D choose to preserve the ship at the expense of the dock and has to pay 4. Concerns regarding Private Necessity a. Even though people can encroach on private property in cases where there is an imminent threat the defense cant be used when it threatens law and order policy implication i. London Borough of Southwark v. Williams (homeless family become squatters)(L): if the ct. permitted homelessness to count as private necessity and use it as a defense to squatting then others who imagined they were in need or invented the need would try to gain Page 21 of 58

Torts Outline Prof. Heymann Fall 2009 entry which would be a threat to law and order; also homelessness may not be viewed as an imminent threat since it is a chronic condition iii. Public Necessity 1. One is privileged to commit an act which would otherwise be a trespass to a chattel or a conversion if the act is or is reasonably believed to be necessary for the purpose of avoiding a public disaster 2. Government does not have to pay for the damages 3. Surroco v. Geary (mayor orders destruction of a house to stop a fire that was raging through the city)(NL): the government has a right to take the necessary step to prevent the fire from spreading II. The Negligence Standard A. Elements of Negligence i. Duty requires the D to conduct himself according to certain standards, so as to avoid unreasonable risk to others ii. Breach failure of the D to comply with their duty iii. Causation Ds breach of duty was the cause of the Ps injury iv. Damages B. Duties and Limitations i. The existence of a duty between parties is a question of law and must be decided by a judge, not a jury ii. People do not start out with duties to one another; a duty must be established before liability can arise iii. General Duty of Care The Reasonable Person 1. When a person engages in an activity he is under a legal duty to act as an ordinary, prudent, reasonable person and it is presumed that an ordinary, prudent and reasonable person would take precautions against creating unreasonable risks 2. There is no duty of care for a person to take precautions against events that cant reasonably be foreseen 3. The Reasonable Person Standard (Rest. 283, pg. 122) a. Unless the actor is a child, the standard of conduct to which he must conform to avoid being negligent is that of a reasonable man under like circumstances b. Note: Reasonable is not the same thing as rational 4. Mental Ability and Mental States a. D must act as would a person of average mental ability; individual mental handicaps (eg. Low IQ) are not considered Page 22 of 58

Torts Outline Prof. Heymann Fall 2009 i. Williams v. Hays (ship captain refused two tugboats aid during storm)(NL): Epitome of the Reasonable person standard Ct. stated that D was bound to exercise reasonable care and prudence to save the ship and D went beyond what was reasonable and prudent by staying up for 48 hrs. to save the ship and the captain cant be expected to do more; the ct. held that the D acted as a reasonable under the circumstances ii. Vaughan v. Menlove (man has a haystack and despite neighbors warning of the fire hazard, sets a fire nearby)(L): man tried to argue that he should not be held liable because he isnt a very smart person; Ct. holds that they cant consider the mental state of every individual because it is too broad and a clear standard cant be created under that ideal; reasonable man standard is still best b. The mental capacity of the P can be taken into account i. Lynch v. Rosenthal (mentally retarded mans arm was caught in a corn picker)(L): D tried to argue that P was contributorily negligent but Ps mental retardation prevented him from understanding the dangers so he will not learn to take greater care; the only way to prevent injury would be for him to stay inside the house which is not an optimal result; he acted reasonably for his mental circumstances ii. Oliver Wendell Holmes stated that when a man has a distinct defect of such a nature that all can recognize it as making certain precautions impossible, he will not be held answerable for taking them c. D is deemed to have knowledge of things known by the average member of the community; the individual shortcomings of the particular D is not considered i. Weirs v. Jones County (man who could not read English, did not understand the Bridge unsafe sign and drove over the bridge)(NL): D should have been literate in English to avoid the bridge; the average member of the community can read English and could have avoided the bridge ii. Friedman v. State (orthodox Jewish girl jumps from ski lift so she wouldnt violate the Jichud)(L): The fear of spending the night with a man on a chair lift w/o means of rescue would be justifiable to an average member of her community 5. Physical Infirmities a. A person is expected to know their physical handicaps and must act with reasonable care under the circumstances i. Kerr v. Connecticut Co. (deaf man walks along trolley tracks; cant hear the warning gong and is struck by the trolley)(NL): a deaf person, who cant hear the warning gong, should have known better than to Page 23 of 58

Torts Outline Prof. Heymann Fall 2009 walk along the tracks; they should have taken reasonable care by keeping a safe distance from the tracks ii. Davies v. Feinstein (blind man used a cane and the felt the walls as he walked down the sidewalk; he fell through an open cellar door)(L): the blind man took the reasonable care by attempting to compensate for the blindness through aids 6. Age a. A child is generally held to that measure of care that other minors of like age, experience, capacity and development would ordinarily exercise under similar circumstances i. Dunn v. Teti (6 yr. old swung a stick at another 6 yr. old)(NL): Ct. provided the following guidelines: i. Minors under the age of 7 yr conclusively presumed incapable of negligence

ii. Minors b/t the ages of 7 and 14 are presumed incapable of negligence, but that presumption is a rebuttable one that weakens as the 14th year is approached iii. Minors over the age of 14 yrs are presumptively capable of negligence, with the BOP on the minor to prove incapacity b. But a minor can be held to an adult standard of care if he is engaging in an activity that is dangerous to others and is normally engaged in by adults i. Purtle v. Shelton (17 yr old. was in the woods hunting deer w/ his 16 yr old friend and accidentally shot him)(NL): It's hard to determine whether deer hunting is an activity normally done by adults because minors do hunt (a minor under the age of 16 can hunt lawfully w/o a license); In this case, the deer hunting does not satisfy both requirements ii. Dellwo v. Pearson (12 yr. old boy ran across mans fishing line with his powerboat; fishing rod to broke and a piece of the reel flew into P's eyes causing injuries)(L): driving a motorboat is a dangerous activity because of the catastrophic results that can occur and it is an activity normally completed by an adult i. Ct. states that there is a policy implication: It would be unfair to the public to hold a minor to a lesser standard when the minor is operating a motor vehicle; the public cant determine if a minor is driving a car, plane or boat and thus can't take the necessary precautions even if warned

c. An elderly individual is held to the reasonable person standard; they are not excused from liability because of their age i. Roberts v. Ring (77 yr. old man who has poor sight and hearing hits a 7 yr. old boy)(NL): Ct. held that the mans physical infirmity due to Page 24 of 58

Torts Outline Prof. Heymann Fall 2009 age does not preclude him from liability; an adult of any age, would have taken reasonable care by staying off the road iv. Duties Arising from Affirmative Acts 1. The law generally imposes a duty of care on people when they engage in affirmative acts which can create risks for others (misfeasance) 2. The law generally does not impose a duty of care on people when they fail to take action to rescue; there is normally no duty to rescue others at the risk of endangering your own safety (nonfeasance) 3. Encouraging or enticing a mentally competent adult into a risky situation does not create a duty of care a. Yania v. Bigan (man jumped into Ds water filled ditch and drowned)(NL): As a mentally competent adult the decedent had the ability to infer from the height of the wall and the murkiness of the water that jumping into the ditch was a bad idea; instead the decedent decided to take on the risk and volunteered to jump so the D should not be liable 4. A D has no duty to an individual unless he created the risk through his affirmative acts a. Weirum v. RKO General, Inc. (Real Don Steele holds contest where listeners have to be the first to reach him and answer a question correctly to win a prize; kids speed to get to him and kill someone)(L): D should have reasonably foreseen that the DJs primary listeners who are kids would have sped throughout to town to get to him and thus endanger others on the road 5. There is no duty to assist in an emergency (when asked) unless there is an imminent threat of bodily injury and the burden is low (remember you have no duty to help others at the risk of your own safety) a. A phone in a public area creates a reliance factor and expectation b. Soldano v. ODaniels (bartender refuses to permit patron to use the telephone to call for help)(L): the phone was in a public part of the building and the patron should have been allowed to use it; it would not have put the bartender at risk to use the phone; the threat c. Stangle v. Firemans Fund Insurance Co. (ring was stolen from women when trying to sell it; receptionist refused to let her use the phone)(NL): the theft a ring is not an imminent threat of bodily harm thus the receptionist was under no duty to allow P with the use of the phone v. Duties Arising from Undertakings 1. Sometimes a D who had no duty to a stranger may acquire a duty by undertaking to provide assistance or otherwise voluntarily assuming responsibilities 2. Negligent Performance of Undertaking to Render Services aka Good Samaritan Doctrine (Rest. 323, pg. 231) Page 25 of 58

Torts Outline Prof. Heymann Fall 2009 a. One who undertakes, gratuitously or for consideration, to render services to another which he should recognize as necessary for the protection of the other's person or things, is subject to liability to the other for physical harm resulting from his failure to exercise reasonable care to perform his undertaking, if i. His failure to exercise such care increases the risk of such harm, or ii. The harm is suffered because of the other's reliance upon the undertaking 3. Duty of one who takes charge of another who is helpless (Rest. 324, pg. 231) a. One who, being under no duty to do so, takes charge of another who is helpless adequately to aid or protect himself is subject to liability to the other for any bodily harm by (a) the failure of the actor to exercise reasonable care to secure the safety of the other while within the actor's charge, or (b) the actor's discontinuing his aid or protection, if by so doing he leaves the other in a worse position than when the actor took charge of him. 4. When a physician begins to hear the symptoms of a patient this signals the physicians undertaking to render the service of medical aid and a duty is created a. Hurley v. Eddingfield (woman sends messenger for medical help; doctor refuses to go see her even though no one else was available)(NL): he had not seen the patient and the messenger did not relay symptoms so there was no undertaking b. ONeill v. Montefiore Hospital (man at hospital relays symptoms to doctor over the phone and was told to go home and return the next day)(L): relaying the symptoms signaled an undertaking; once you begin to render aid you have to follow through and you cant pull out 5. Once an individual undertakes to render services they must do so with reasonable care; failure to use reasonable care will result in liability a. United States v. Lawter (inexperienced Coast Guard crew member tried saving a woman during a storm)(L): the crew undertook the service of saving the woman and failed to use the reasonable care of putting an experienced crew member on the sling b. Frank v. United States (crew tugs boat and a man on the boat falls into ocean and drowns)(NL): the crew originally undertook the service of tugging the boat and thus came equipped for that purpose; they only had the proper equipment for the tow and they tried to use reasonable care (given the circumstances) to save the man 6. When you begin to render assistance to an individual, you cant discontinue the aid and leave the individual in a position worse than they were before you assumed control a. Octillow West Joint Venture v. Superior Ct. (Man tells golf course that he will drive his drunk friend home and once they leave sight of the Page 26 of 58

Torts Outline Prof. Heymann Fall 2009 employees allows the drunk friend to drive; drunk friend dies in a crash; family sues golf course)(NL): the man assumed control by offering to drive the drunk friend home and deterred the employees efforts to prevent drunk friend from driving; his interference put the drunk friend in a worse position than when the employees had the keys b. Cuppy v. Bunch (After drinking during a fishing trip man tells his friend, who claimed he wasnt feeling well, to follow him home in his car)(NL): The man did not undertake aid/assume control over his friend or the car compared to Octillo where the man took the car keys vi. Special Relationship 1. The special relationship between parties can impose a duty when a duty would not normally exist 2. Duty to assist others a. Common Carriers have a duty of care to assist their passengers and prevent foreseeable injuries that occur while transporting the passenger to a reasonably safe place b. Brosnahan v. Western Air Lines (Passenger is hit over the head by another passengers luggage)(L): airline has a duty to supervise the boarding process and the airline is responsible for the foreseeable risk of being hit in the head if there is not proper supervision c. Boyette v. Trans World Airlines (drunk passenger gets off connecting flight and steals a golf cart; a chase ensues and he hides in a chute that leads to a trash compactor and dies when its activated)(NL): decedent had reached a reasonably safe place so the airlines duty to him ended; the P could sue the airport and claim a duty but it is not foreseeable that a passenger would climb into a chute 3. Duty to protect others from third parties a. General Principle (Rest. 315, pg. 239) i. There is no duty so to control the conduct of a third person as to prevent him from causing physical harm to another unless i. A special relation exists between the actor and the third person which imposes a duty upon the actor to control the third persons conduct, or

ii. A special relation exists between the actor and the other which gives to the other a right of protection b. There is no duty of care to warn an individual of danger/threat except in cases where there is a specific and precise threat from the third party i. Tarasoff v. Regents of the University of California (physician did not warn decedent when his patient made threats to kill decedent)(L): the special relationship/duty arose from the patient/doctor privilege the Page 27 of 58

Torts Outline Prof. Heymann Fall 2009 doctor has a duty (as outlined by the AMA) to break confidentiality to protect the welfare of an individual ii. Thompson v. County of Alameda (juvie offender with violent propensity towards children threatened to kill a kid in the neighborhood when released to moms care; killed neighbors kid 24 hrs. later)(NL): the county did not have a special relationship with the victim and the threat was not to a particular individual; a warning to the general population would be inefficient and ineffective i. My thoughts is that this ruling was wrong - it would have been efficient and effective for the police to warn the parents of small children within a few blocks radius of where the offender lived; when looking at this in the hand formula the burden of notifying this select segment of the population would have been low and the PL would have been high considering his violent nature and the warning itself

ii. Policy Implications for the offender warning neighborhoods would jeopardize rehabilitative efforts because of the stigma surrounding criminals c. There is a duty to protect an individual from harm of a third party when a D has exclusive control over the premises where an injury occurs (pertinent for landlord/tenant situations and hotels) i. Kline v. 1500 Massachusetts Avenue Corp. (tenant assaulted and robbed in the hallway of her apartment complex)(L): landlord had exclusive control over the hallways and had exclusive power to take the necessary security precautions; landlord also had notice of previous assaults so he should have taken reasonable care to ensure tenants safety d. The police do not have a duty to protect a specific individual unless they can prove a special relationship (eg. Police beginning to render aid through witness protection); the duty extends to public at large i. Riss v. City of New York (woman goes to police stating her ex is threatening to kill her but they do not offer protection; ex later has lye thrown in her face)(NL): The amt. of protection to be provided is limited by the resources of the community and they can't afford to protect her based on threats of harm 4. Special Relationships also include: a. School/student b. Parent/child c. Employer/employee d. Innkeeper/guests e. Business that holds its premises open to the public Page 28 of 58

Torts Outline Prof. Heymann Fall 2009 vii. Duties Arising from the Occupation of Land 1. Duties to Trespassers a. Trespassers one who comes onto the land without permission or privilege b. General Rule i. Generally no duty, except duty of reasonable care in carrying out activities risking serious bodily harm for known trespassers (w/ duty to warn of highly dangerous conditions in some jurisdictions) ii. Duty to avoid inflicting injury by "willful or wanton" conduct in some jurisdictions iii. Don't have a duty to go out and look for trespassers on your land but once you see them you have to take care iv. Haskins v. Grybko (landowner mistook a trespasser for a woodchuck and shot him)(NL): P cant be held liable when he did not know that trespasser was on his land; cant be expected to take reasonable care when you dont know an individual is on your land v. Herrick v. Wixom (trespasser sneaks into circus and is injured by a stray firecracker)(L): Even though the circus did not know he was a trespasser, he was a member of the audience and the circus had a duty to use reasonable care to protect its audience; when you see a person sitting in front of you, you have to use reasonable care, regardless of their status vi. Ehret v. Village of Scarsdale (gas leak from a pipe under the street; gas seeped into a vacant house where trespasser slept)(L): Similar to circus case, it didn't matter that the decedent was a trespasser; he was part of the general public and the D should have taken care to make sure he wasnt harmed; the gas like the firecracker was a danger to the public as a whole, not just to the individual c. If its foreseeable that a trespasser will step onto a Ps land, the P must take reasonable care in carrying out activities risking serious bodily harm for known trespassers (w/ duty to warn of highly dangerous conditions in some jurisdictions) i. Cleveland Electric Illuminating Co. v. Van Benshoten (trespasser uses outhouse that sits over a sewer)(NL): It was not foreseeable that a man would walk into a construction site to use an outhouse, thus the P did not have a duty to warn the D with a sign d. Attractive Nuisance Doctrine i. If a condition on the land is reasonably foreseeable risk that would attract the attention of children, the D will be liable i. Keffe v. Milwaukee & St. Paul R. Co. (P had a RR turntable that kids leg got caught in)(L): Ct. held that D knew Page 29 of 58

Torts Outline Prof. Heymann Fall 2009 the turntable revolved easily, that it was attractive to kids, that many kids lived in the area and had a habit of playing and that the turntable was dangerous to kids when put into motion; D should have taken reasonable care (eg. put up a fence) to protect the children when they could not be expected to protect themselves 2. Duties to Licensees a. Licensee one who enters on the land with the landowners permission, express or implied, for her own purpose or business rather than the landowners benefit; social guests are considered licensees b. General Rule i. Duty to warn of dangerous hidden conditions of which owner is aware (and licensee is unaware) ii. Duty of reasonable care in undertaking activities iii. No duty to inspect c. The relationship between the parties are important to determine if the D is a licensee or an invitee i. Davies v. McDowell National Bank (couple visit wifes stepfather at his office; wife and stepfather died from carbon monoxide poisoning)(NL): Ct. had to determine whether the wife was a social guest or a business visitor; ct. held that she was a social guest since he was her stepfather and the ct. believed it did not matter that the incident occurred in the stepfathers office; since the wife was social guest/licensee then the D can only be liable if he knew of the latent dangerous condition and failed to take reasonable care; P couldnt show that D knew of the condition 3. Duties to Invitees a. Invitees one who enters onto the premises in response to an express or implied invitation of the landowner and includes those who enter onto premises opened to the public (eg. Museums) and those connected with the business (eg. Store customers); business visitors are considered invitees b. General Rule i. Duty of reasonable care both as to condition of property and as to activities ii. Duty to warn and duty to inspect iii. Jacobsma v. Goldbergs Fashion Forum (man grabs thief at a store when manager yells Stop Thief)(L): store had knowledge that thief tried to steal clothing three days earlier and it was foreseeable that he would do so again; since the man was a business customer, the D owed him a duty to protect him from the thief Page 30 of 58

Torts Outline Prof. Heymann Fall 2009 c. Firefighters and police officers are treated as licensees because society gives them an implied consent to enter the premises even when an owner may not be on site to grant permission; firefighters and police cant recover for risks present on the property, if the owner is unaware of the risk d. Scope of Invitation a person loses their status as an invitee if they exceed the scope of the invitation and goes into a portion of the premises where her invitation cant reasonably be said to extend (eg. Customer goes beyond the employees only door) e. Some jurisdictions challenge the distinctions between trespasser, licensee and invitee and instead state that you must treat non-trespassers with reasonable care i. Rowland v. Christian (womans guest cuts his hand on a broken faucet which she asked the landlord to fix a month before the incident)(L): Ct. held that an individuals status has some bearing on the question of liability but it is not determinative ii. Carter v. Kinney (member of a Bible study group went up to a house for a meeting and slipped on ice)(NL): Ct. disagrees with Rowland and the trend to get rid of the distinctions; Ct. holds that the distinctions balance the interests of the injured and the possessors of land; the distinctions also create fairly predictable rules so that entrants and possessors can determine appropriate conduct and juries can assess liability iii. "Duty" doctrines serve as a way of controlling juries since a ruling that D has no duty or a limited duty is usually a determination of law to be made by a judge, as opposed to decision about whether a D exercised reasonable care which would best be answered by jury as a determination of fact (breach) viii. Negligent Infliction of Emotional Distress (NIED) 1. This is not a separate tort; it is a way of talking about a type of cases in which the question is whether the D had a duty to refrain from carelessly causing the emotional distress; the harm is the emotional distress in the negligent case; it is at root a negligence case 2. To claim NIED there must be a physical manifestation of the mental suffering/injury (eg. Depression or anxiety) 3. Pandoras Box Problem a policy implication a. With NIED, cts. Are concerned that if they establish liability, they will be opening up suits to a huge class of Ps b. The cts. want people to be able to recover but they also want some limitations to ensure people arent taking advantage of the system 4. There are two types of NIED cases: zone of physical danger cases and bystander cases Page 31 of 58

Torts Outline Prof. Heymann Fall 2009 5. Zone of physical danger cases are those in which the P is the direct victim of the negligence (eg. near miss cases) a. Robb v. Pennsylvania Railroad Co. (woman was almost killed by a train when her car got stuck in a rut on the tracks)(L): i. Impact Rule i. In the past, cts. Believed that NIED could only be claimed if a physical injury also occurred

ii. Cts. Believed fright alone did not give rise to a cause of action, so the consequences of fright will not give rise to a cause of action iii. The physical consequences of fright are too remote and that the requisite causal connection is unprovable iv. Fright is a subjective state of mind and is difficult to evaluate, easy to prove for P and difficult for D to disprove; this will lead to the opening of Pandora's box, where the cts. Will be inundated with claims ii. Ct. held that the Impact Rule is discredited and an individual can recover for NIED if they are within the zone of danger i. It is now generally agreed that the gist of the action is the injury flowing from the negligence, whether physical impact or nervous shock

ii. With the advance of medical science, the resulting injury can be traced back to the fright iii. Jurisdictions that allow this cause of action has not been flooded by those cases and if it is then the jurisdiction will just have to cope; cts. Deal with claims of pain and suffering based on subjective symptoms all the time b. "Zone of physical danger" is a useful limitation for NIED because the fear and harm would be understandable if you are in the zone compared to someone who sees the accident from a safe distance c. But cts. Place limitations on who can use the zone of physical danger argument to limit the amount of possible Ps i. Lawson v. Management Activities, Inc. (a plane crashed near a Honda dealership and as it went into its dive, the employees at the dealership believed it was going to crash into them)(NL): Duty of care imposed on airplane operators does not extend to emotional distress of people who were physically untouched by a plane crash, even if they feared it would crash into them; holding the plane liable would open up Pandoras box and there would be a huge class of P's claiming NIED since a lot of people would see a plane crash Page 32 of 58

Torts Outline Prof. Heymann Fall 2009 ii. Quill v. Trans World Airlines (plane went into a nosedive and pilot regained control 5 seconds before the plane hit the ground)(L): assigning liability on the D would not open Pandoras box because this case deals with a smaller class of Ps (passengers on the plane) d. Toxic Tort cases i. In this sub-category of zone of physical danger cases, a P may claim that they have been exposed to a toxic chemical and fear that their exposure could result in illness ii. To prevail they must show that they were exposed to the chemical and that there must be a probability that they will contract the illness i. Potter v. Firestone Tire and Rubber Co. (Ps lived next to a landfill where the D dumped toxic carcinogenic chemicals, despite the govt telling them this action was illegal)(NL): Ct. held that policy concerns require the P to show that there is a probability of getting cancer (eg. Burden of payment of awards based on fear alone would be borne by the public in higher insurance premiums or businesses going without insurance)

6. By-stander cases are those in which someone else is the victim of physical injury and the P witnessed the injury a. Factors Ct. consider with bystander cases: how closely related the bystander was to the victim, whether the bystander actually saw the accident occur and how close the bystander was to the accident b. Relationship between bystander and victim i. If there is a close relationship between the bystander and the victim (eg. Mother/son; husband/wife) then the bystander could recover ii. Barnhill v. Davis (man saw his mother hit by a car when she was driving behind him)(L): It is reasonably foreseeable that a son would suffer mentally and a jury can decide whether a reasonable person would believe his mother would be seriously injured or killed by the accident c. Temporal proximity i. If the bystander was on site, when the accident occurred then they could recover ii. Marzolf v. Stone (fathers son was killed in a motorcycle accident and the father came up on the scene 10 minutes after the accident occurred)(L): the trauma of seeing a loved one on the scene of an accident stems not only from seeing the accident occur but also from witnessing the aftermath of the accident d. Spatial Proximity

Page 33 of 58

Torts Outline Prof. Heymann Fall 2009 i. If the bystander was nearby when the accident occurred, they could recover ii. Gain v. Carroll Mill Co. (father saw footage of his son being killed on the evening news)(NL): Mental suffering of a relative who is not present at the scene of the accident is unforeseeable as a matter law and thus the D cant be liable e. A D cant be held liable if a P mistakenly believes someone is injured by an accident i. Barnes v. Geiger (woman saw someone get hit by a car and thought it was her son)(NL): Distress based on mistake will vary from person to person based on their ability to imagine the worst that can happen; to extend liability to individuals who are distressed based on mistake would add an unreasonable amount of people a person can be liable to C. Breach i. A D breaches their duty when their conduct falls below the level of reasonable care owed to the P ii. The Hand Formula is used to determine whether that conduct falls below the level of reasonable care 1. United States v. Carroll Towing Co. (employees on a tugboat did not properly retie the lines on the barge; barge floated away and was struck by a tanker propeller and sunk; bargee was not onboard)(L): Ct. created the hand formula and held that the bargee was liable for the damage because the burden/cost of having the bargee on board would be low (since he was supposed to be on board in the first place) and the probability of a serious harm resulting from this lack of proper supervision was high Hand Formula B = the burden/cost of taking adequate precautions P = probability of the harm occurring as a result of untaken precaution L = the gravity of the resulting injury B>PL = not negligent it would cost more for the D to prevent the accident so the D should not have to take the precaution/care and thus they are not negligent; instead the burden is put on the P; this is what basically happens in an accident

B<PL = negligent it would cost less for the D to prevent the accident so the D should take the precaution; this assumes that the precaution will successfully reduce the L; it's possible, that the precautions may not be successful 2. Adams v. Bullock (boy swings a 8ft. long wire which comes into contact with trolley wires under the bridge)(NL): B = putting the wires underground P = the ease of touching the trolley wire from the bridge (not high because the wires were under the bridge and you can't reach it even if you lean over) Page 34 of 58

Torts Outline Prof. Heymann Fall 2009 L = electrical shock B>PL = not negligent 3. Bolton v. Stone (cricket ball went over a fence and hit a neighbor in the head)(NL): B = the cost of preventing balls from going into the street P = the likelihood that a ball hits a person (low because only a few balls have come over the fence in the past 30 yrs.) L= the harm caused by being struck by a ball B>PL = not negligent 4. Eckert v. Long Island R. Co. (man spotted a child sitting on RR tracks and pushes kid out of the way but is struck by the train himself and later dies)(L): D tried to argue that the man was contributorily negligent; ct. holds that negligence implies some act or omission that is wrongful and there is nothing wrong with a man saving a childs life; if the man believed that he could save the child without harm to himself then he cant be negligent B = not going onto the tracks to save the child when it is definite that the child would die (the decedents B includes the PL of the child) P = the likelihood that he would die (if he had stayed off of the track then the likelihood would have been miniscule) L = value of his own life PL(B)>PL = man is not negligent iii. Custom and the Problem of Medical Malpractice 1. Customs help to determine what is considered reasonable care 2. Even though a D may take the precautions that are standard or customary in their field, a ct. can rule that the precaution breaches a duty, if the standard is not an example of reasonable care 3. Evidence of compliance with custom or non-compliance with an industry custom is relevant and admissible, but not determinative a. The T.J. Hooper (tug boat did not have working radios on board and there could not hear warnings of an approaching storm; lost barges in the storm)(L): D argued that working radios were not an industry standard so they should not be held liable; Ct. held that radios are a cheap and easy means of avoiding storms so they should keep up with the times and get the working radios B = cost of getting radios P = probability of injury in a storm L = injury can be high if its a serious storm B<PL= negligent b. Ellis v. Louisville & Nashville Ry. (RR worker inhales dust for 25 yrs. And sues claiming that the D should have provided him with masks to prevent him from getting silicosis)(NL): Ct. held that there have been no injuries from the Page 35 of 58

Torts Outline Prof. Heymann Fall 2009 lack of masks so there is nothing wrong with the practice; in this case, custom was a good defense 4. Custom is only relevant when the conduct/standard in question is not inherently dangerous; if it is, then the custom will have to change a. MacDougall v. Pennsylvania Power and Light Co. (plumber goes on roof on a rainy day and hit his head on an unmarked fuse box that sat on the roof edge; he was electrocuted)(L): Ct. held that the placement of a fuse box near a roofs edge was an inherently dangerous conduct and the industry standard must change; a high degree of danger always calls for a high degree of care 5. If the parties have a contractual relationship, the reasonable standard of care is fixed by the markets and tort law ensures that companies comply to these standards a. Rodi Yachts, Inc. v. National Marine, Inc. (unattended barge slipped free after being in port for a few days and crashed into 2 other boats)(unknown): Injurers have to worry that if they endanger their customers they may lose them or have to charge a lower price to compensate them for bearing the risk of injury; As a result the market fixes its own standard of care to reflect the preference of potential victims and potential injurers; both parties have to minimize their liability costs by allocating responsibility for safety measures efficiently whether explicit or implicitly by abiding by the customs of the market 6. Medical Malpractice a. Custom is dispositive and the standard of care a doctor was bound by either falls under the locality or national rule b. Locality rule i. Old Point of View i. Doctors were bound by the standards that were prevalent in the community in which they practiced Doctors should be bound by the standards that are prevalent in communities that have similar geography and medical facilities as the community in which they practice a. Gambill v. Stroud (woman had complications from anesthesia which resulted in cardiac arrest and brain damage)(NL): It has not been shown that the same postgraduate medical education, research and experience is equally available to all physicians regardless of where they practice; if you look at communities with similar demographics you will get a more accurate depiction of the proper standard of care ii. Proponents support this rule because education and facilities vary tremendously from place to place and an allowance Page 36 of 58

ii. New Point of View i.

Torts Outline Prof. Heymann Fall 2009 should be made for the country doctors inability to keep up with his city counterpart iii. Locality rule (not the national rule) applies to facilities or services a hospital may provide i. Johnson v. Wills Memorial Hospital & Nursing Home (man escaped hospital and died of exposure)(NL): protection is not the function of a hospital but instead is a service that the hospital provides; this service will be limited by the resources a hospital has available so the locality rule is appropriate

c. National rule i. National rule Doctors are bound by the standard of care of the average practitioner ii. This is beneficial since medical education is becoming more uniform nationally iii. Brune v. Belinkoff (woman in a small town outside of Boston was given 8mg of an anesthetic and she felt numbness from the drug; a Boston physician would have given her 5mg)(L): Ct. held that the locality rule is absurd in this day and age; the skill of the doctor should take into account the advances in the profession iv. Negligence Per Se 1. Negligence is a means of establishing the standard of care necessary to determine breach 2. Negligence Per Se when a D violates a statute that creates a standard of care, the violation establishes the D negligence as a matter of law a. Martin v. Herzog (man hits a couple, who is riding in buggy, with his car on a dark highway; man does not have his lights on)(L): The main issue in this case was whether or not the man had his lights on; Highway law states that lights are used for guidance and protection of travelers and to willfully or heedlessly omit them is to fall short of the standard of care enacted by the statute therefore the man breached his duty and is negligent b. Tingle v. Chicago (mans cows are hit by train on a Sunday)(NL): P sues claiming that D violated the statute that no trains would operate on Sunday; the statute was created for religious reasons not for the creation of a standard of care and as a result there is no breach of duty 3. The statute has to be related to the claim a. Brown v. Shyne (patient injured by unlicensed chiropractor)(unknown): the statute was suppose to protect people from risk of injury by unskilled and careless doctors; it did not create a standard of care that is related to this claim; the patient was injured because the D didnt have a license, he was injured because the D was negligent; P should have used under regular negligence Page 37 of 58

Torts Outline Prof. Heymann Fall 2009 4. A D cant use contributory negligence as a defense in a negligence per se case because they breached the standard of care the law established a. White v. Levarn (man shoots his friend when they went hunting on a Sunday)(NL): even though P was technically contributorily negligent (by voluntarily going hunting), it still does not change the fact that D broke the law by hunting 5. Exception to the negligence per se rule - If complying with a statute poses a greater danger to an individual than non-compliance, the individual will not be found liable for violating the statute a. Tedla v. Ellman (brother and sister violate a state law requiring them to walk along the road, facing the traffic and they are hit by a car)(NL): If they complied with the law, the brother and sister would have been walking on the more heavily trafficked side of the street; the statute was enacted to protect pedestrians and if they followed it, they would have been placed in greater danger; the legislatures do not want to put people in more danger so violation was acceptable v. Res Ipsa Loquitur (the thing speaks for itself) 1. Remember - to make the argument that D violated a duty of care, P points to some action that D should have taken to prevent injury 2. Ps who cant easily point to an action, can use RIL as another tool to show breach 3. RIL places an extreme burden on the D a. A P has the burden to prove their claim and if they dont then they lose b. Technically a D can sit at a trial and do absolutely nothing to counter the Ps claim c. in RIL the D has to prove they were not negligent 4. Elements of RIL a. The thing that happened to the P must be the type of event that ordinarily does not occur except through the negligence of someone b. The instrumentality of the harm suffered by the P must have been w/in the D's exclusive control c. The harm to the P must not have resulted from any conduct on the part of the P 5. Byrne v. Boadle (man is hit with a barrel of flour that fell out a warehouse window)(L): Epitome of RIL barrels don't fall out of windows everyday so there has to be some sort of negligence involved; even though the D may not have been the individual to drop the barrel, the barrel came from his warehouse and was under his exclusive control 6. RIL does not occur without negligence Page 38 of 58

Torts Outline Prof. Heymann Fall 2009 a. Combustion Engineering Co. v. Hunsberger (workman drops a tool at a construction site that hits another construction worker on the head)(NL): the probability that a tool will fall is high and can occur even when an individual is not acting negligently b. Judson v. Giant Powder Co. (there was an explosion at a powder factory where dynamite is manufactured)(L): Resp. provided expert testimony which stated that if the correct process of manufacturing and handling of the dynamite was taken carefully, then an explosion would not have occurred; since an explosion occurred it's probable that the D did not take the proper care c. Compliance Errors and RIL i. Compliance error momentary failure to take repetitive precautions (eg. Looking both ways before crossing the street) ii. Compliance errors increase as technology makes an activity unusually safe and people begin to rely heavily on this technology iii. Accidents in areas with the most safety equipment/better technology are the strongest RIL cases because on the rare occasion when something goes wrong its usually because of negligence i. Haasman v. Pacific Alaska Air Express (plane disappears without a trace during a flight)(L): because no evidence was left behind, both sides are clueless as to the exact cause of the disappearance; but a plane ordinarily does not disappear unless there was some negligence involved

iv. Accidents in areas with less safety equipment/low technology are weaker RIL cases because accidents occur more regularly and its likely not due to operator negligence i. Walston v. Lambertsen (crab boat sank and crew drowned; parties arent sure of the cause of the accident)(NL): despite Ps argument that a newly installed crab tank was the cause of the sinking its technology is so primitive that there are many hazards that can lead to its destruction without anyone being negligent

7. Exclusive Control a. Larson v. St. Francis Hotel (woman hit in the head with an armchair as she walked along the sidewalk outside of a hotel)(NL): A hotel does not have exclusive control of its furniture - guests have at least partial control; the hotel cant take ordinary care to prevent an accident like this from happening because a guest is most likely to blame for such an accident occurring, not the D b. Connolly v. Nicollet Hotel (woman hit in the eye with mud-like substance as she walked along the sidewalk outside of a hotel; members of a Page 39 of 58

Torts Outline Prof. Heymann Fall 2009 convention were being destructive and hotel issued a memo to employees about the destruction)(L): The hotel knew that the guests were destructive; they had exclusive control over whether the guests should be there and the proper course of action would have been to throw them out 8. Ruling out Ps contribution to the injury a. Wilson v. Stillwill (man has surgery on his arm and the arm becomes infected and eventually paralyzed)(NL): P offered evidence that showed infection rates at the hospital were below the national average but this does not suggest that when an infection does occur it occurs because of negligence; its very possible that the infection came from the patients improper care of the wound 9. Issue of Multiple Defendants a. In cases where there are multiple defendants the P is not precluded from recovery just because they cant point a finger at a particular individual b. They can get recovery if they can show that the probability of injury was caused by the negligence of at least one of them i. Ybarra v. Spangard (man had a surgery where multiple doctors and nurses were involved; the muscle between his neck and shoulder became paralyzed)(L): Every D had a duty to exercise ordinary care towards the P and would be liable for breach of this duty; Also w/ so many people involved in a surgery and the fact that the P is unconscious, it is unreasonable to expect the P to identify the individual who caused the injury; D tried to argue that there was a division of labor so the Ds did not have exclusive control over the instruments of harm but this argument failed since they were all working together as a team ii. Wolf v. American Tract Society (there were 19 independent contractors at a site; brick fell and hit a man on the head)(NL): Since there were numerous independent contractors and none of them were subject to the control and direction of the other, you have to be able to identify the actual injurer; you cant just choose to only hold 2 Ds accountable (when there are 19 other possible Ds) without any evidence; Policy Implication Sometimes you're going to have situations where you cant identify or prove the injurer and in those cases it is better to let the injury go w/o redress then punish individuals who may not be the injurer c. A contract between Ds can demonstrate that both had exclusive control i. Bond v. Otis Elevator Company (woman injured when elevator went into freefall)(L): Since there was a contract between the Ds that showed they owned and maintained the elevator, they have exclusive control Page 40 of 58

Torts Outline Prof. Heymann Fall 2009 d. Some could argue allowing a P to sue multiple Ds without any direct evidence places is unfair for the D; they are being sued even though a P its equally possible another person is the negligent party D. Cause i. Causation draws the link between breach and damages ii. But-for Causation and Proximate Causation are necessary to determine whether or not the D caused the injury complained of by the P iii. Cause in fact 1. Cause in fact the Ds conduct was the actual cause of the injury 2. But for test a. Primary means of proving cause in fact and is the traditional view of causation b. P has the burden to show that the untaken precaution would more likely than not have prevented the injures (51% threshold) c. But for the Ds negligent act, the injury would not have occurred i. New York Central R.R. v. Grimstad (man who cant swim falls overboard; wife tries to save him but there is no lifesaving equipment on board)(NL): P argues that the untaken precaution = lifesaving equipment; there was no evidence that the proper equipment would have saved him; for all we know, he still could have drowned; also the man couldnt swim so that could have contributed to his death d. Cons of the Traditional View i. The traditional view supports an all or nothing approach if the chance of survival was 51% or higher then the D can be liable; if it was below 51% then they wouldnt be liable ii. This approach subverts the deterrence objectives of tort law by denying recovery for the effects of conduct that causes statistically demonstrable losses

iii. It also creates pressure to manipulate and distort other rules affecting causation and damages in an attempt to mitigate perceived injustices iv. It gives certain defendants the benefit of an uncertainty which, were it not for their tortious conduct, would not exist

3. Loss Chance of Survival a. When a P has difficulty proving that the Ds conduct was the actual cause of the injury, they can use loss chance of survival to argue that Ds conduct decreased their chance to survive and resulted in harm i. Gardner v. National Bulk Carriers, Inc. (man goes overboard and no one realizes it for 6 hrs.)(L): D argued that man was in the water for 6 hrs. and it was moonless night so it would be near impossible to find Page 41 of 58

Torts Outline Prof. Heymann Fall 2009 him/he would most likely be dead; ct. holds that seaman can live in the water for hours and their refusal to turn around eliminated any possibility of the man being found alive; this is in comparison to Grimstad where the Ds actions could not prevent death b. Depending on the jurisdiction, a D can be found liable even if the decedents original chance of survival was below 50% i. Herskovits v. Group Health Cooperative of Pudget Sound (doctors did not diagnose a patients cancer and it was found too late; if caught it earlier decedent had a 39% chance of survival; when they caught it later he had a 25% chance of survival)(L): If cts. Found for the D it would permit a blanket release from liability for doctors anytime there was a less than 50% chance of survival; doctors must try their best even if the chances aren't good (policy consideration) ii. Dumas v. Cooney (doctors did not diagnose a patients lung cancer and it was found too late; if caught it earlier decedent had a 67% chance of survival; when found later he had 15-20% chance of survival)(NL): There has to be some degree of certainty that a D did cause the P's injury; If the acts of the D did not actually cause the P's injury then there is no rational justification for requiring Ds to bear the cost of P's damages iii. Dillon v. Twin State (boy electrocuted when he fell off bridge and grabbed a high voltage wire)(unknown): D is liable for the small portion of conscious suffering, he endured from the moment he touched the wire until he fell to the ground 4. Alternative Liability a. In cases where there are 2 Ds, the P may have a difficult time pointing to the particular individual who caused the harm (remember Ybarra) b. If its clear that both Ds brought about the negligent situation, both parties will be held liable; D has the burden of showing that the other defendant caused the injury i. An opponent may argue that it is unfair to hold a person liable when they may not be the cause of the injury ii. A proponent may argue that is more acceptable to hold 2 people liable when there is a 50/50 chance that they caused the injury compared to a suit involving 5 Ds where there would be 20% chance per person iii. Summers v. Tice (P and 2 Ds went quail hunting; when Ds shot at the bird, P was struck in the face by bird shot)(L): Using Ybarra as precedent, ct. holds that Ds can be liable for damage whether they are deemed to be acting in concert or independently 5. Market Share Liability Page 42 of 58

Torts Outline Prof. Heymann Fall 2009 a. Market share liability is prevalent in defective product cases where there are three or more Ds b. If it is clear that they all brought about the negligent situation, liability can be split among the parties based on their percentage of the market (the more you contributed, the more liable you are) c. If a D can prove that they did not make the product that contributed to the Ps injuries then they will not be found liable d. Sindell v. Abbott Laboratories (woman had a problem identifying which company manufactured the drug that her mother took during pregnancy which caused an illness)(L): each companys drugs had the exact same formula so we know they sold a dangerous drug; D is just trying to hide behind the fact that the exact batch can't be traced to this individual; if the evidence shows that the Ds comprised 90% of the market then the probability of getting the guilty parties is high e. Opponent POV i. MSL gives preferential treatment to people who can't trace the D who caused the injury; unlike other Ps, they are allowed to seek recovery against an entire industry or segment of the injury ii. Holding an entire industry responsible is going to increase economic costs 6. Joint and Several Liability a. When it is impossible to tell which D caused part of the injury, the P can collect the all of the damages from either D b. The D who paid the damage could not seek reimbursement from their codefendant c. Joint and Several Liability isnt prevalent anymore; instead cts. Make Ds responsible for only a portion of the damages and that portion reflects their responsibility for the injury iv. Proximate Causation 1. You can have a causation chain that extends to far-reaching and bizarre consequences (eg. Girl breaks up with boyfriend he tries to impress her and buys a van full of flowers the van explodes flowers onto the highway obstructing another drivers view other driver crashes into a pole he sues the girl) 2. Proximate cause is a tool the cts. Use to draw a line to ensure it is holding the necessary people responsible and keeping out extraneous Ds 3. Direct causation and foreseeability are two opposing views used to examine proximate cause but foreseeability is becoming the prevalent view and may become the sole criterion for determining proximate cause 4. Direct Page 43 of 58

Torts Outline Prof. Heymann Fall 2009 a. The direct view holds the D liable for all consequences that flowed directly from their negligent acts regardless of whether the consequences were foreseeable b. In re Polemis (gas leak on a ship; a plank falls causing an explosion and the destruction of the ship)(L): plank fell because of negligence and the plank was a direct cause of the explosion; Unforeseeability is irrelevant 5. Foreseeability a. The foreseeability view holds the D is liable for foreseeable consequences of their negligent acts i. Holding an individual responsible for unforeseeable consequences is unfair With negligence we ask whether a reasonable man would have seen the risk and decided not to take it; if something is unforeseeable then a reasonable man could not determine whether or not the risk should be taken ii. The Wagon Mound (No.1) (oil spills into bay; two days later the oil caught on fire and destroyed ships and docks)(NL): It is not fair that a D be liable for an act of negligence which has unforeseeable consequences, even if the act is a direct cause of the damage b. It does not matter if the consequence is a rare occurrence i. The Wagon Mound (No.2) (oil spills into bay; two days later the oil caught on fire and destroyed ships and docks)(NL): Ct. holds that D knew it was possible that the oil in the water could ignite (rare, but possible) and no reasonable man would dismiss such a risk c. It does not matter if the D did not foresee the extent of the damage i. Petition of Kinsman Transit Co. (ice loosened boat from the dock which struck other boats, brought down a drawbridge and flooded the banks)(L): It is not unreasonable to hold the D liable when the same level of care that would have prevented the small foreseeable damage (boat colliding with another boat) would have prevented the large unforeseeable damage (knocking out a bridge) ii. Colonial Inn Motor Lodge v. Gay (man backs into a heating unit; thinking he caused no damage he drives away; leak causes an explosion)(L): Ct. holds that the impact was substantial and that the possibility of damage from hitting a heating unit is not so farfetched that is freakish or fantastic 6. Intervening Causes a. Intervening cause comes into the picture after the D has already set things in motion with their negligent act; it doesn't operate concurrently (ie D sets a fire in the area negligently and the wind is blowing when the fire is set, it's not an intervening cause because it's a background condition) Page 44 of 58

Torts Outline Prof. Heymann Fall 2009 b. An intervening cause does not automatically free the D of liability; a D is free of liability when the intervening cause is a superseding cause c. General Rule: If an intervening cause is foreseeable the D is still liable i. Brauer v. N.Y.Central & H.R. R. Co. (train hit man on wagon; his goods spill on the road and thieves take the goods)(L): D had guards on the train to protect their goods showing that the theft was foreseeable; since it was foreseeable the D should have taken care to prevent the Ps goods from being stolen d. Third Party Intent i. If a third party acts with intent and the intent was unforeseeable, then the D is not liable ii. If a third party acts without intent (eg. negligence or accident) and this was foreseeable then D is liable iii. Watson v. Kentucky & Indiana Bridget & R.R. (train derailment causes gas leak in city)(unknown): The issue was whether Duerr ignited a match with the intention of starting a fire; if gas spills into the street then it's probable and foreseeable that some random individual will light a match which would cause an explosion; but the D could not have foreseen that someone would light a match purposefully with the intent to cause an explosion iv. Alexander v. Town of New Castle (gambler throws the man who arrested him into a pit in the street)(NL): COMPLIES WITH (i); ct. holds that the gambler acted with intent and it is unforeseeable that someone would purposefully push an individual into the pit, so the D is not liable v. Village of Carterville v. Cook (man is jostled and falls off of 6 ft. sidewalk)(L) : COMPLIES WITH (ii); ct. holds that the individual who jostled the P did not do so with an intent to harm and jostling on a sidewalk is foreseeable so the D is liable e. Where Ds actions cause another to react instinctually, D will be liable for any harm inflicted by the reacting person to another i. Scott v. Shepard (man tosses firecracker in market; patrons fling it away from them; takes out one persons eye)(L): the blame lies with the first thrower of the firecracker; you can't blame the individuals who flung it away to protect themselves as they were doing it for their own safety 7. Limitation of Liability a. Liability can be limited by either limiting duty or limiting causation i. Limiting duty Was the injured party owed a duty by the D in the first place? Page 45 of 58

Torts Outline Prof. Heymann Fall 2009 ii. Limiting causation - Was the Ds activity the proximate cause of the injury? b. Palsgraf v. Long Island R.R. Co. (passenger jumping on train drops unmarked package of fireworks; passenger on platform injured)(NL): i. Limiting duty (majority) Cardozo i. The RR owed the P a duty to make sure she was safely transported and could be held liable for any injuries that extend from that duty

ii. The do not owe her a duty in regards to the package since they had no idea what was in the package; if they are unaware of the risk, then they cant take the necessary precautions to ensure her safety iii. The owner of the package owes her a duty since he knew what the package contained and would understand the risk to others ii. Limiting causation (minority) Andrews i. Andrews claims that there is a duty between the RR and the woman but proximate cause is a better means of limiting liability

ii. A duty analysis is a matter of law and wouldnt be decided by the jury iii. Also by allowing the decision to be made by the jury liability is determined on a case by case basis compared to make a sweeping generalization/categorization that could exclude a lot of cases from even going to trial iv. By looking at it based on duty you will understand which group you're responsible to and which you aren't compared to Andrews where you can be liable to anyone iii. Differences b/t two views: Judge vs. jury; case by case vs. categorization; broader social justice vs. individual justice E. Damages i. Functions 1. Achieving corrective justice by compensating P for losses resulting from Ds tortious act 2. Promoting deterrence by threatening Ds with liability for costs resulting from their tortious acts 3. Law governing damages in tort law is unavoidably linked with the purposes underlying the imposition of tort liability itself ii. Compensatory Damages Page 46 of 58

Torts Outline Prof. Heymann Fall 2009 1. P recovers damages to compensate her for the losses proximately resulting from the Ds tortious act or omission. 2. Damage to Property a. The damages are measured by the loss in value of the property or the cost to repair (replace), whichever is less. b. The exchange value of property is the market value at the time of the loss, or if no market exists, the value of something similar i. If the subject matter cannot be replaced (as in a family portrait), compensation is for its special value to P (original cost, quality and condition at the time of loss) c. U.S. v. Hatahley - (Feds. Seized Indian tribes horses) i. Assessed loss of animals according to market value (replacement cost); not sentimental value i. Administrative costs: everyone has a certain attachment to the things they have, impossible to assess

ii. Causation: how much of loss was accorded to governments actions and how much would have happened anyway: some of these sheep would have died anyway; full value for all sheep isnt fair 3. Personal Injury/Death a. P is entitled to recover to repair Ps injury or make him as whole as may be done by an award of money: b. Economic losses and expenses i. P is entitled to recover all economic losses and expenses she has suffered as a result of the injury (includes past and present expenses) i. Medical expenses

ii. Loss of consortium/society loss of companionship as a result of a decedents death iii. Lost earnings and earning capacity (subtract taxes but add benefits) a. Factors to be considered: i. Occupation/Education ii. Health/Age iii. Dependents

Page 47 of 58

Torts Outline Prof. Heymann Fall 2009 b. If a person who seeks recovery for their deceased spouse gets remarried, they can still recover i. Benwell v. Dean ii. Landers v. Ghosh - (carpenter awarded 400k): decedent was 22 yr. old unemployed; had GED; was a smoker; married; 1 child iii. Pescatore v. Pan Am - (CEO awarded 9 million-loss of support, 5 million-loss of consortium): Harvard Physics Grad, MBA from Univ. of Chicago; married; 33 iv. Haddigan v. Harkins - (Housewife awarded 60k): calculated how much her cooking, cleaning, etc. costs; D argued that the cost of maintaining the decedent should be included in the calculation v. Louisville & Nashville Ry. V. Creighton - (Death of 4 year old child): kid has no earning capacity and because of his age any calculation of lost earnings is too speculative c. Pain and Suffering Awards i. The Functions of the Award i. In the case of serious injury, compensation for out-of-pocket loss alone could never make the P whole

ii. P may be able to use an award of general damages to provide activities or enjoyments that substitute for those lost as a result of the injury a. Olin Corp. v. Smith (teenager with amputated leg awarded $5.58 million): 16 yrs old with life expectancy of 55.8 yrs.; endured months of pain while doctors attempted to save leg; phantom pain b. Williams v. US - (prisoner with amputated leg awarded 500k): 48 yrs old with a life expectancy of 24 years; prisoner w/ history of drug and alcohol use; some phantom pain ii. Limitations of Awards i. Criticism subjectivity means significant variations

ii. Some states have ceilings to increase predictability and decrease possible unfairness iii. Punitive Damages Page 48 of 58

Torts Outline Prof. Heymann Fall 2009 1. Punitive damages serve as a punishment to deter a D and other parties from committing a similar tort in the future 2. Cons (Murphy v. Hobbs) a. By allowing punitive damages, you are in essence prosecuting a D twice for a single tort since a D can be indicted and prosecuted in criminal ct. for an intentional tort b. By prosecuting the D twice you're violating his right against double jeopardy c. It is unfair for P to receive compensation for the physical injury, any resulting mental injuries and a wrong committed to the public at large i. Possible Solution to this problem: i. All punitive damages goes to a fund ii. some states allow split recovery between the fund and the P iii. But this could backfire - if the jury wants more money to go to the P, they'll increase the money awarded d. Punitive damages are problematic with gross negligence cases; luckily there can't be any criminal prosecutions but the cts use very guarded language to limit the damages 3. Pros (Kemezy v. Peters) a. Compensatory damages are objective so the awards may fall short; by falling short Ds conduct may be not be deterred b. Punitive damages on the other hand can deter some conduct c. Compensatory damages will not deter people who derive pleasure from the tortious conduct or those who can conceal the conduct (eg. Are caught only 50% of the time); punitive damages may deter d. Punitive damages express the community's abhorrence of the D's act e. Punitive damages relieve pressures on the criminal system; also if there weren't punitive damages people would take matters into their own hands for minor issues because they believed their complaints have been ignored (pretty thin reason to me) 4. To ensure that punitive damages are not too large, they must be evaluated based on the following (Grimshaw v. Ford Motor Co.) a. Is the sum so large as to raise a presumption that the award was the result of passion and prejudice and therefore excessive b. Does the award bear a reasonable relationship to the D's net assets? c. Does the award bear a reasonable relationship to the compensatory damages awarded? 5. Future financial prospects are relevant in punitive damages (Rufo v. Simpson) Page 49 of 58

Torts Outline Prof. Heymann Fall 2009 6. Punitive Damages and Employers a. Punitive damages can be awarded against an employer for the conduct of an employee if but only if: i. The employer authorized the conduct and the manger of the conduct or ii. The employee was unfit and the employer was reckless in employing him or iii. The employee was employed in a managerial capacity and was acting in the scope of his employment or iv. The employer ratified or approved the act b. Remember - the purpose of punitive damages is to punish the wrongdoer and you should punish the employee who committed the tort, not the employer, unless there was some deliberate corporate participation c. Kennan v. Checker Cab Co. (cab driver pulls blind man with seeing eye dog out of his car and beats on him)(NL): there was no evidence that the employer knew of or authorized its drivers to forcibly eject, assault or batter their customers; liability rests solely on the driver d. In re Exxon Valdez (drunk captain left bridge; oil tanker hit reef and spilled 11,000 gallons of oil in the ocean)(L): Exxon knew the captain was an alcoholic and that the crew's work schedules recklessly created a large risk of fatigue thus showing the employer was reckless in its decision to employ the captain and the employee was unfit III. Strict Liability A. Strict liability focuses on activities where we say the nature of the activity is such that we want the D to pay whether they're at fault at not. B. Differences between regular liability and strict liability Regular Liability Applicable in cases where anyone and everyone can be a potential P or D Strict Liability There is a small number of potential Ds because only a small group of people are going to engage in certain activities The D will not take all precautions because it's a waste of money when they will be held accountable regardless of how much effort they take. It will not result in the D making any changes in their behavior. Instead by making them pay all of the cost you want them decide to refrain from the activities in general.

Normally we ask for a D to take reasonable care. This will not prevent all accidents from occurring because it is not cost effective. But by imposing regular liability you will do not prohibit people from engaging in the activities, you just force them to do it carefully/do it in a different way

Page 50 of 58

Torts Outline Prof. Heymann Fall 2009 C. Liability for Animals i. A person who keeps an animal with knowledge (scienter retinuit) of its tendency to do harm is strictly liable for the damage that occurs if it escapes; person has an absolute duty to confine or control the animal to prevent injury and liability will result for failure to control ii. Animals that are ferae naturae are animals that are by nature harmful or have not been tamed and domesticated; they are presumed to have a tendency to do harm and scienter doesn't have to be proved 1. Behrens v. Bertram Mills Circus, Ltd. (circus elephant frightened by dog; tramples midget couples booth)(L): The D is liable for any injury the elephant did while out of control; Ct. points out that there would be no liability if the injury occurred when the elephant tripped or stumbled 2. Candler v. Smith (baboon escapes from zoo and chases woman into her house)(L): baboon is a ferae naturae, the P doesnt have to allege negligence in allowing the animal to escape because the owner is bound to keep it secure; The mere fact that it escapes, regardless of how, is enough for the D to be liable iii. Animals that are mansuetae naturae are presumed to be harmless until they manifest a savage or vicious propensity; proof of this manifestation is necessary to determine liability (usually based on animals prior bad acts) 1. Earl v. Van Alstine (man kept beehives near public highway; bees attack passerbys horses and kills one)(NL): Ct. considers bees to be completely domesticated; since they were kept on property for 8 or 9 yrs. w/o injury and the fact that they did not exhibit a savage or vicious propensity, the D will not be held liable; Note: the ct.s desire to designate the bees as domesticated may be a policy implication if you hold D liable then a lot of others people would be liable which may deter people from keeping bees 2. Smith v. Pelah (L): Ct. held that if an owner has notice of a dogs previous conduct of biting then he can be held liable if the dog bites someone again iv. D will be liable for the injury caused by an animal if the injury is within the scope of the risk (eg. elephants trampling things if not under control vs. woman just tripping over boa constrictor she let's roam her house) 1. Bostock-Ferari Amusements v. Brocksmith (man walks muzzled bear peacefully down the street; another mans horses were frightened and injured him)(NL): the bear was walking along peacefully and the injury did not result from some vicious propensity of the bear thus the petitioner cant be liable D. Abnormally Dangerous Activities i. Elements 1. In determining whether an activity is abnormally dangerous, the following factors are to be considered: a. Existence of a high degree of risk of some harm to the person, land or chattels of others; Page 51 of 58

Torts Outline Prof. Heymann Fall 2009 b. Likelihood that the harm that results from it will be great; c. Inability to eliminate the risk by the exercise of reasonable care; d. Extent to which the activity is not a matter of common usage e. Inappropriateness of the activity to the place where it is carried on; and f. Extent to which its value to the community is outweighed by its dangerous attributes ii. Rylands v. Fletcher (men built reservoir on their land; water floods into neighbors coal mines)(L): Ct. held that creating a reservoir, put the land to a non-natural use; this activity was not an example of common usage and the risk was foreseeable because the nature of water is to flow wherever it can; even though the Ds did not know about the tunnels its foreseeable that it would cause some sort of damage iii. Miller v. Civil Constructors, Inc. (man hurt by a stray bullet at a rural firing range frequented by police)(NL): Risk of harm of firing a gun can be eliminated with reasonable or utmost care in many circumstances; the use of firearms is common and the harm comes from people's misuse rather than a guns inherent dangerous nature; Location was most likely appropriate to reduce the risk of injury to people; the target practice had social utility since it was done by policemen iv. Social Value Criterion 1. The social value criterion has caused a controversy for two reasons: a. The decision on the utility and value of an activity is subjective and controversial b. It is illogical to say that the cost should be borne by a P when the activity has economic value; a harm has been created and the one responsible for such harm should bear the cost regardless of utility 2. Lubin v. Iowa City (city left its underground water pipes in place until they burst and damaged P's property)(L): Pipes have a social utility so you dont want the city to stop laying the pipes, you just want them to stop the policy so you make the policy strictly liable; if the ct. holds them liable under negligence they may try to take reasonable care in regards to the policy, which the ct. does not want 3. The social value criterion is not dispositive and an extremely valuable enterprise may be found liable for the dangers if they are sufficiently great a. Indiana Harbor Belt Ry. Co. v. American Cyanamid Co. (train car with toxic chemical leaks at yard in city)(NL): D cant be held strictly liable when taking reasonable care is enough to prevent the accident; you wouldnt want to hold the D strictly liable because you still want companies to transport chemicals, you just want them to do it safely so this case should be brought under negligence not strict liability E. Respondeat Superior (Let the master answer) i. Respondeat Superior holds an employer strictly liable for the torts committed by their employees in the course of their work (aka scope of employment) Page 52 of 58

Torts Outline Prof. Heymann Fall 2009 1. Does not preclude a negligence suit against the employer alleging that they breached a duty in hiring, supervision or management of the employee; the doctrine simply permits a suit where no such showing needs to be made to hold the employer liable 2. Does not preclude a tort claim against the employee ii. Who is an employee? 1. If the one securing a service (ie the company) controls the means by which the task is accomplished, then the one performing the service is an employee 2. If not, then they are an independent contractor and the company cant be held liable under respondeat superior 3. Miami Herald Publishing Co. v. Kendall (paperboy runs over man)(NL): employee had sole discretion at choosing how he delivered the papers; he had the choice of how the task the accomplished iii. Scope of Employment 1. A tort is within the scope of employment if the tortfeasor is acting with an intent to further his employers business purpose even if the actions are indirect, unwise or forbidden 2. Ira S. Bushey & Sons v. United States (drunk seaman floods drydock and ship falls against a wall)(L): the sailor lives where he works, and is required by his profession to return to the dock while intoxicated; this falls under the scope of employment 3. Forster v. Red Top Sedan Service (driver yells at couple no old bastard would delay his schedule and hold him up from getting to the beach and punches them in the face)(L): employees statement shows that he was acting for a business purpose 4. Reina v. Metropolitan Dade County (bus driver chases man down after man gives him the finger)(NL): he acted from anger after getting the finger, not from furthering his employers business iv. Frolic and Detour 1. The cts. Distinguish between a frolic and a detour to determine where the employers liability ends 2. Frolic the employee pursues personal business and this qualifies as a substantial deviation from or abandonment of the employment 3. Detour employees deviation is still sufficiently related to the employment to fall within the scope of employment a. If its foreseeable that an employee will deviate somewhat (usually determined by the physical distance of the deviation) then the employer may be held liable (eg. Employee runs to bank for his boss and picks up lunch on his way back in)

Page 53 of 58

Torts Outline Prof. Heymann Fall 2009 4. Miller v. Reiman-Wuerth Co. (man gets permission from supervisor to run to bank; on his way back he gets into an accident)(NL): employee left to conduct personal business so this does not fall within the scope v. Commuting to Work 1. In general, employers arent responsible for torts committed while an employee is commuting to work 2. Exception when the employers conduct creates an extra risk to the public which will manifest itself during the commute (eg. Drinking) 3. Konradi v. United States (postal service in rural area required mailman to use their personal cars for delivery)(L): this requirement meant the mailmen couldnt use public transportation or carpooling where available and forced them to be on the road; this increases the chances of an accident occurring IV. Defenses A. Contributory and Comparative Negligence i. Contributory Negligence 1. D argues that the P failed to take reasonable care in their own safety; this defense if proven, is a complete bar to recovery regardless if D is shown to have some liability a. Harris v. Meadows (P, who had the right of way, hits D who failed to make a successful left hand turn)(P = L): a reasonable person would have slammed on the break, instead of just slowing down, when they see a car cross their path 2. Last Clear Chance doctrine a. A P could recover despite committing contributory negligence if the D had a sufficiently good opportunity to avoid the accident at a point when the P did not b. P typically had to show that the D saw the P or had some other notice of him but failed to avoid inflicting injury c. Reasons for this exception to contributory negligence i. Later negligence of the D involves a higher degree of fault ii. Ps negligence is not a proximate cause of the harm to him because the Ds actions is a superseding cause which relieves the P of responsibility iii. Also, seems to be a response to dislike of contributory negligence d. Davies v. Mann (P left donkey by the side of a highway; D ran over donkey)(P = NL): even though the donkey shouldnt have been there, the D should have been going at a slower speed; if had then he would have noticed the donkey earlier and could have taken care

Page 54 of 58

Torts Outline Prof. Heymann Fall 2009 3. Contributory negligence has been abandoned by most states and replaced with Comparative Negligence ii. Comparative Negligence 1. Pure Comparative Negligence - reduces the P's damages by the percentage of negligence; permits recovery even if Ps fault is greater than D 2. Modified Comparative Negligence - P's recover based on the percentage if: a. Ps negligence does not exceed the Ds (50% or more) or b. Ps negligence is less than the D's (49% or less) 3. McIntyre v. Balentine (D's tractor collided with P's pickup truck; P was drunk and speeding)(P = L): Ct. decides to adopt 49% modified comparative negligence iii. If a P has engaged in unlawful conduct, courts will not entertain suit if Ps conduct constitutes a serious violation of the law and the injuries for which P seeks recovery are the direct result of that violation 1. Manning v. Brown (girls steal a friends car and goes joyriding; D hits a pole)(P = L): there was no duty b/t the girls and their relationship arose from their unlawful action 2. Fritts v. McKinne (man gets into accident from drunk driving and hes brought into hospital; D sued for negligently cutting artery during surgery)(P=NL): there was a duty b/t the doctor and patient (undertaking to render aid) and the car crash and malpractice are not directly related Smith (P) Costs incurred Liability Contributory Negligence Damages awarded Costs incurred Pure comparative negligence Damages awarded Costs incurred Modified comparative negligence Damages awarded Costs incurred B. Express Assumption of Risk i. Express Assumption of Risk P explicitly agrees to assume the risk of injury and frees D of liability Page 55 of 58 $100,000 51 percent $0 (damages) Jones (D) $100,000 49 percent $0

$100,000 (costs $100,00 incurred) $49,000 $102,000 $0 $151,000 $51,000 $98,000 $51,000 $49,000

Torts Outline Prof. Heymann Fall 2009 ii. Questions to ask: 1. Did the P understand what they were agreeing to? 2. Even if they did, should the contract not be enforced because of policy reasons? iii. To enforce the contract there must be: 1. Exculpatory Clauses a. Exculpatory clauses protect the D from suits arising from their own negligence b. Van Tuyn v. Zurich American Ins. Co. (woman signs waiver to ride mechanical bull; requested speed not be too high)(L): agreement was too broad; D should have protected itself with the exculpatory clause because without it the P probably didn't understand what she was giving up 2. Equal bargaining power a. To determine if there is equal bargaining power the ct. examines the following: i. The importance of the subject matter to the physical and economic well-being of the P ii. The amt. of free choice the P could have exercised when seeking alternative services b. Manning v. Brown (man sky dives and has parachute malfunction)(NL): skydiving is not necessary for physical or economic well-being and the P could have gone elsewhere to skydive or decided to forego the activity altogether i. Policy Implications i. The sky diving industry may go out of business if the cts. don't enforce the contract

ii. Enforcing this agreement will decrease the cost by the D so the next time someone sues the D can show the P that it is enforceable so you shouldn't bother litigating iii. Also it keeps the D's insurance costs down by having this agreement iv. If you enforce these agreements are you basically just letting people off for their own negligence? Doesn't that defeat the point of negligence suits if you can just sign away your rts? c. Anderson v. Erie Ry. Co. (clergyman bought reduced fare tickets; the back said, if you purchase this ticket you assume all risk of injury)(NL): P could easily have bought the regular ticket if he wanted the regular protection/duty being a passenger entails; there was equal bargaining power iv. Waivers of liability are unenforceable if they involve public interests Page 56 of 58

Torts Outline Prof. Heymann Fall 2009 1. Contracts are deemed to affect the public interest and will be invalid if: a. The contract covers a transaction which concerns a business generally thought suitable for public regulation and holds themselves out to render a service to members of the public b. The party asking for exculpation has a bargaining advantage against any member of the public who seeks their service c. The person is placed under control of the party seeking exculpation and is subject to the risk of carelessness of that party 2. Tunkl v. Regents of the University of California (man signed hospital waiver for negligent acts; later sues for malpractice)(L): Contract falls under each category listed above and the patient has no bargaining strength; they have no choice but to submit to the release and that is unfair C. Primary and Secondary Assumption of Risk i. Primary Assumption of the Risk 1. D had no duty to protect the P from the harm suffered because the risk of it was inherent in an activity that the P chose to undertake; the inherent activity entails certain risks that P is aware of and decides to ignore 2. Determination of liability depends on the following: a. The openness of the risk is it hidden or misrepresented b. the voluntariness of the Ps participation in the activity in question c. the reasonableness of the risks posed by the activity in light of its benefits (P may know that an activity entails a particular/certain type of risk) 3. Murphy v. Steeplechase Amusement Co. (P got on The Flopper and was injured)(NL): the name of the ride and the fact that the P's wife and the other members of their party saw people falling on the ride showed that they knew there was a risk of injury; they decided to take the risk so the D is not liable 4. Woodall v. Wayne Steffner Productions (human kite gets bad driver)(L): P was not aware of the certain risk that he was given an inexperienced driver so he did not assume the risk 5. Cohen v. McIntyre (woman takes dog to get neutered; he bites vet after muzzle is removed)(NL): his profession entails the certain risk of being bitten (its inherent in the activity) and by that profession he assumes the risk 6. Ps do not assume the risks of negligent conduct by those they have no relationship with a. Neighbarger v. Irwin Industries (Ps safety employees injured in petroleum fire, when Ds employees negligently released a valve)(L): i. Firefighters rule - free people from a duty for a class of people whose employment arises from being in dangerous situations; the public is paying to be exonerated from this duty through their taxes and it would Page 57 of 58

Torts Outline Prof. Heymann Fall 2009 be unfair to also have them pay for injuries resulting from the firefighters job ii. Unlike the relationship b/t the firefighters and the public, D did not pay for the services of the safety employees thus they did not assume the risk 7. This is not a true defense because the P cant est. a prima facie case because there isnt a duty ii. Secondary Assumption of Risk 1. D had a duty to P because the risk was not inherent in the activity but P knowingly and voluntarily chose to encounter the risk (eg. Need to get to the hospital during an emergency and volunteer to get in a car with a drunk driver) a. Marshall v. Ranne (man attacked by neighbors vicious hog)(L): P had no choice between being attacked by the hog to cross his own properly or being shut up in his house; Ps choice of the former was not taken voluntarily b. Kennedy v. Providence Hockey Club (woman sitting in fourth row gets hit in face with hockey puck)(NL): woman has seen countless games in person and on TV to know the risk; just because these were the only tickets she could get, doesnt mean she was stripped of all choices; she could have decided to forego the event c. Hennessey v. Pyne (woman is hit by golf ball while examining her garden)(L): the risk was not inherent in the activity of examining her flowers; the only way for her to avoid being injured would be stay in the house which (similarly to Lynch) is not an optimal result 2. Unlike primary assumption of risk, this is a true defense 3. Secondary Assumption of Risk and Comparative Negligence a. In many jurisdictions secondary assumption of risk has merged into comparative negligence and the question comes down to the reasonableness of the Ds behavior compared to the P b. If the P acted reasonably then he recovers the damages regardless of whether he understood the risks and chose to encounter them c. If the P acted unreasonably, then his damages are reduced

Page 58 of 58

You might also like